ProfResp Barnett Fall10 Cases

Published on January 2017 | Categories: Documents | Downloads: 46 | Comments: 0 | Views: 259
of 68
Download PDF   Embed   Report

Comments

Content

Missed Oct 17

 Session 1.1
 In Re Nunnery y Overview: Attorney was suspended for, inter alia, violating Wis. Sup. Ct. R. 20:1.1 by failing to make any meaningful inquiry into the veracity of suspicious documents his client gave him as the documents were obviously fraudulent and Supreme Court of Wisconsin was not persuaded attorney was, as he implied, merely a hapless victim of an unscrupulous client. y Facts j Sole practitioner, without previous discipline problems j T.T: ED dispute, reached agreement with N on fees, but was never reduced to writing  Failed to file a response after requesting two extensions, and complaint was dismissed. T.T. tried to contact N but was told he was gone. Convinced her to file a federal suit, which T.T. sent a $200 cheque to N who put it in his own files and not into client trust account. (never returned check and did not file the federal suit«check never negotiated)  On 2nd suit against employer, employer responded, but N/T.T. never replied (Which was decided to be a no probable cause determination)  Referee found Nunnery never intended to reply, but only to conduct discovery for federal court claim. T.T. then lost a right-to-sui by not filing, and T.T.¶s claim was barred. y VIOLATIONS With T.T. i. (M.R. 1.5.c)- Failing to reduce the 60/40 contingency basis in writing whereas the contingent fee must be in writing, denoted expenses that are deducted, and the recovery if any. ii. (MR 1.15.a)- keeping property of clients separate from the attorney's property, that is to say, against comingling; D failed to keep $200 check in trust account, kept it in a file iii. MR 1.3 - Reasonable diligence, whereas Defendant failed to respond to ERD deadlines resulting in a dismissal iv. MR 1.4(a) (Clauses 3 + 4) - Failure to Communicate with client about status, meaning tha thte lawyer must reasonably inform client about status of matter and comply with reasonable requests for information v. MR 1.4b - Communication to client to the extent reasonably necessary to permit the client to make informed decisions regarding their representation, whereas Defendant never intended to pursue the ERD claim With J.A. i. Knowingly advanced a claim not warranted by law w/o good faith argument, violating RULE 3.1 that a lawyer shall not knowingly advance a claim or defense that is unwarranted under existing law, except that of which is in good faith. With D.D i. Rule 1.4(b) not communicating strategy w/ D.D., failed to explain to extent reasonably necessary to permit client to make informed decision regarding the representation-----still did not file federal complaint. Did not explain strategy to D.D. about his strategy of using one suit to get discovery on another suit. (D.D. tried to contact N but with not luck, but later was told everything was on track. N failed to tell D.D. case was on hold). N told D.D. he would refund retainer, but never did. ii. Rule 1.3 failing to act w/ reasonable diligence (not performing work on D.D¶s case)







Rule 1.4a failing to respond to requests for information, and assuring that everything was on track, not explaining N was putting case on hold for reason, and telling him he had mailed the check. Not keeping client informed. With E.J. i. 1.1: failure to provide competent representation. (failure to meaningfully inquire into the veracity of the suspicious e-mails and letters that his client claimed were sent by defendants in the EJ case; blindly relying on client¶s unreasonable assurances that the documents were authentic, and that their extraordinary content were actually reduced to writing by alleged authors; by ignoring sworn statements by original authors that documents were fabrications, arguing the wrong legal standard, presenting an appellate brief with only one page of argument. The referee found misconduct on this count (10) not b/c N failed to discover his client¶s fraud, but because he failed to make any meaningful inquiry into the veracity of the suspicious documents. (described as obviously fraudulent) N argues that 1.1 doesn¶t apply b/c he intended to protect clients only. But the court says 1.1 is intended to protect the system of justice as well as individual clients. Further, his suggestion that his clients misdeeds relieve an attorney of this obligation to serve and protect the legal system is rejected. ii. 3.1/4.4(a) took actions which he knew or was obvious that it was merely to harass or maliciously injure another. With Y.P i. 1.3: failed to act w/ reasonable diligence and 1.4, (failure to keep clients reasonably informed). Took up a 3k retainer check, but nunnery was non-communicative, and case was dismissed. He continued evading her communications about refund and legal situation. Improperly filed a complaint on their behalf. Did not inform Y.P of the judgment which constituted a lien against her real estate. ii. V. R i. 1.1 failed to provide competent representation to client, was told to move for a substitution of parties but did not, and did not move for extension. y 1.1: A lawyer shall provide competent representation to a client. j Poor brief writing j Failed to inform client about the costs she owed, thus creating a lien against her property. j He had taken on too much and the result was a disciplinary action«he was a very competent attorney, but the demands were just too much. Conclusion: 2-month suspension plus requirement pay back E.J for the sanctions imposed on her. Holdings: The Supreme Court held that: (1) attorney's conduct in filing employment discrimination lawsuit on behalf of client in federal court, without making any meaningful inquiry into veracity of purported e-mails, letters, and memos from employer's personnel to client, which documents client had provided to attorney, violated professional conduct rule requiring competent representation, and (2) two-month license suspension, with reinstatement conditioned on attorney's payment of Rule 11 sanctions which federal court had imposed on attorney for filing frivolous employment discrimination complaint, was appropriate disciplinary sanction for attorney's failure to provide competent representation to two clients, lack of diligence in representing three clients, failure to communicate with three clients, and other misconduct. Model Rule Violations: y Rule 1.15 Safeguarding Property (c) A lawyer shall deposit into a client trust account legal fees and expenses that have been paid in advance, to be withdrawn by the lawyer only as fees are earned or expenses incurred. j Nunnery took check for $200 from client (for fed. Filing fee to be paid) and placed it in clients folder not in bank and he failed to file in federal court.

iii.

y

y

y

y

Rule 1.3 Diligence ± A lawyer shall act with reasonable diligence and promptness in representing a client. j Nunnery failed several deadlines resulting in the statute of limitations being up and/or dismissal of cases. Rule 1.4 Communication (a)(3) A lawyer shall keep the client reasonably informed abut the status of the matter. j Nunnery didn¶t return phone calls and was unavailable to his clients as their cases progressed. Rule 1.4 (b) A lawyer shall explain a matter to the extent reasonably necessary to permit the client to make informed decisions regarding the representation. j Nunnery failed to explain to client TT that he never intended to pursue a complaint. Court found that Nunnery violated a preamble to the Model Rules that his conduct was an attack on the system of justice. j His client forged documents and the Court said that he should have questioned her more and found that the documents were fakes.

a. Hypos: y Your client is convicted of selling drugs and sentencing has not yet taken place. She tells you that she had been recently arrested for DUI and gave the police her friend¶s ID. At her sentencing for the drugs do you have to disclose that her sentencing report is inaccurate? Yes if the judge asks you during sentencing but probably not otherwise. 1. The VA Ethics Committee says the lawyer should do nothing because their main duty is to defend their client. 2. Rule 1.8 (b) and Rule 3.3 y Lawyer fakes his death leading his wife to believe that he died. He hides for five weeks and then PO¶s find him. He is brought up on ethical violations. His defense is that it shouldn¶t matter because it¶s a private matter. Should he be disciplined? YES. Dishonesty/Misrepresentation. y Can a lawyer be disciplined for having an affair? NO. Unless it was another lawyer who was representing an opposing client. 1. Rule 8.4 (c) & (d) B. Residency Requirements a. ISSUE: Is the residency requirement to practice law in a state a violation of the Article IV Privileges and Immunities clause? b. Privileges and Immunities clause guarantees that non-residents of a state are treated fairly y State government can argue that P&I is not applicable because there is no discrimination against non-residents

 Session 1.2
Rule 8.4: Misconduct -prof. misconduct for a lawyer to © engage in conduct that involves in dishonesty«. (d) the administration of justice« (has an enormously broad reach). Comment 3: What may a lawyer disclose to the spouse of a client (in the hypo, where she's dead) y Attorney-client privilege Rule 1.6 y Comment 2 y 5th line (encouraged to seek«) y Maintain Confidentiality  DC bar (in hypo) took position that lawyer was going to have to make a decision and he was going to have to act with the information he gathered when his client was then alive. y 1.6(b) has exceptions to confidentiality (pg22)

y

B3: probably doesn't apply b/c lawyers services have not been in furtherance of a fraud committed by the wife«.

y

y y

Rule 4.1 (a) lawyer can't say something to husband that is flat-out wrong. (ie she's starting up a business etc«) cannot misrepresent the situation. (b) --> how is this juxtaposed against 1.6 Can't disclose a material fact but can't say something that's wrong. y Wife's conduct isn't fraudulent (if you look up 1.0 it's defined), but (b) concludes Preamble on pg 3 (in relation to hypo with law firm advising on American workers and foreign workers) y Paragraph 5 y Paragraph 9 (pg 4) --> nature of law practice« Explain the ramifications of the crime perpetrated but having done this, if she did not do this, the lawyer had to keep his mouth shut. Rule 3.3. Candor toward the tribunal y "is there anything omitted to this report"? --> Hypo y Life-threatening hernia on a child. Surgery is performed. All is well. y Down road, new client says the same doctor has performed malpractice. Would you represent client? Be impartial?
y y

What if it opens up liability for lawyer malpractice Hypo: Atty involved in extra-marital affair. Wife learned. Lawyer faked death. No representation of client, his acts were outside the court of law. It was a personal matter that had ramifications for the practice of law. 8.4c 8.4d By faking his death he communicated his opinion of the legal profession. 8.4 y 8.5(a) (regardless of where it occurs) Practicing law while suspended«may result in disciplinary actions not only in that state, but also other states where you're permitted to practice as well.

 Session 2.1 ± Admission to Bar (Character Inquiries: greater scrutiny)
 ³Shattered Glass Hypo´ o While on the staff of The New Republic in the 1990¶s, Stephen Glass, a gifted write who also wrote for the Times magazine and other publications, fabricated all or parts of 27 stories. He was in his mid-20¶s. o In response to suspicions, he lied to editors. He created false evidence (documents, a website). o When discovered and could no longer lie, he confessed and broke down. o All the while he was a night law student at Georgetown y Should Georgetown have let him graduate or should it have kicked him out or forced him to take a leave? o Glass has since passed the NY bar exam, but has not been admitted to practice although he has applied. o He¶ll never again work as a journalist. Should he be allowed to work as a lawyer? y Yes- people can change; he should be admitted in X years depending on what he¶s done in the interval. y No- clients and courts will be asked to trust him, unaware of his history. Equivalent conduct as a lawyer would get him disbarred.   In re Mustafa

 While a student at the law school of the University of California at Los Angeles, John Mustafa used his access to the checking account of the school¶s moot court program to write checks for his personal use. After a fellow student discovered and reported his misconduct to the dean, Mustafa confessed his actions to a law school professor and the District of Columbia¶s Committee on Admissions. He also informed the law firm that ultimately hired him. After performing an investigation, the university placed a letter of censure in Mustafa¶s confidential student discipline letter. Mustafa passed the July 1991 bar exam and applied for admission to the Bar of the District of Columbia. In light of his past misconduct, the District¶s Committee on Admissions held a hearing to determine whether Mustafa should be admitted to the bar. The Committee found that Mustafa always intended to repay the money he took, and the Committee was impressed by Mustafa¶s honesty and forthrightness before the Committee and during university investigation. Several school and work acquaintances also gave testimony of Mustafa¶s good character. In a unanimous decision, the Committee recommended that Mustafa be admitted to the Bar. Nevertheless, while noting Mustafa¶s good law school record and appropriate conduct since the embezzlement, the D.C. Court of Appeals held that the short amount of time that had passed since Mustafa¶s misconduct prevented him from establishing good moral character: While we do not hold as a matter of law that an applicant for admission to the Bar, like a disbarred attorney, must necessarily wait a minimum of five years from the date of proven misconduct before applying for admission to the Bar, we conclude that on the record here, particularly the relatively short period of time that has elapsed since the date of his misconduct, Mustafa has failed to establish that he has the good moral character required for admission to the Bar. However, the court believed it likely that Mustafa would be able to establish the necessary good moral character for admission at some point in the future.  Did not show good moral character--> misappropriated funds.  applicant denied admission to D.C. Bar two years after misconduct based largely on recency of misconduct  Other cited grounds for Delaying or Denying Admission to the Bar? y Criminal Conduct (prager: denied b/c of large-scale drug business; nwr, buying stuff in roommates name) states examine nature of crime, how long ago it occurred, and the applicant¶s conduct thereafter. y Lack of Candor in Application Process y Dishonesty or Lack of Integrity in Legal Academic Settings. y Mental Health y Financial Probity y Applicant¶s Private Life y Ability to Speak English.  Barbarian @ The Gates (hale) MR 8.1 pg 125 Applicant for admission to the bar«(a)shall not make a false statement, (b) shall not make a false statement © and knowingly respond falsely. Did his failure to disclose make a false statement of fact? Dishonesty with civility or honesty with confrontational beliefs 8.4B Professional misconduct for a lawyer to commit a criminal act that reflects adversely«. With regard to 8.4b, there is no need for a conviction or a prosecution. Standard of reasonable doubt does not apply to bar matters. 8.4C -professional misconduct for a lawyer to engage in conduct engaging in misrepresentation, deceit, etc«. 8.4D - professional misconduct that is prejudicial to the conduct of justice. y Message of hale case: avoid controversial issues.

 Session 2.2: Admission To The Bar/ Transient Lawyers + Multijurisdiction

 Geographical Exclusion: Supreme Ct. of New Hampshire v. Piper  RULE: A state may not limit attorney admissions to its residents.  FACTS: Piper was a Vermont resident living near the NH border. NH limited attorney admissions to residents of NH only. Piper took the NH bar and passed. She applied for an exemption. This was denied and she sued.  Occupation of lawyer is a privilege protected by the P&I clause. P&I in Sec. 2 of Const. Right must be central to the economic integration of the states. Court found that it does  There is no specific criteria between what violates P&I and what doesn¶t violate. It¶s arguable on either side. y NH ARG: Non-Local attorneys are less likely to be familiar with local laws ad procedure, behave ethically, be available and do pro bono work. j The Ct. said there was no proof of this ARG.  Pg541: we want to help lawyers cross state borders b/c it will help advance certain rights (see Flynt's dissent discussing how much progress was made with out-of-state lawyering. )   400 yards and the Rehnquist dissent- lawyers are part of the governance apparatus of the state. State has an interest in having lawyers there to participate in the governance of the state. y Should a state be allowed an exemption form the privileges and immunities clause that would exempt y Ends up not doing so, even if practice of law is a privileges. y Second step in analysis: Is the test of constitutionality in the P&I clause«does NH have a substantial reason for discriminating against out-of-state residents. There must be a SUBSTANTIAL basis upon which the state is discriminating. y Ct: says NH didn't bring the factual basis required. Which means it is possible that another state MIGHT actually prevail«but the attorneys will have to be better at showing the evidence. y NH attorneys argued based on reason, rather than some statistical, economic, or similar foundation to back their claims.  Tolchin pg 546: Challenge to NJ req. that to practice law in state one must have an office, a bona fide office in the state, and must take a course in skills and methods before you can represent clients.  Third circuit took the opinion that there is no discrimination against non-residents. B/c if there is no discrimination, the clause is inapplicable. Ct says these requirements cover a resident of NJ as well as non-resident. They're not imposing greater requirement against non-residents, but that all attorneys that pass the NJ bar must have an active office in state and must take the course. y NJ requires non-resident lawyers practicing in NJ to maintain an office in NJ and to take a course in NJ on NJ law. y This was upheld after being challenged as a violation of the P&I clause. y ARG for State: State gov. can argue that P&I is not applicable because there is no discrimination against non-residents ------------------y Rule 8.4 y Rule 5.5b1b-and 5.5(c) and 5.5(a) y "other law" can encompass state law and commerce law. y Is there now b/c of repetition of these commercials a continuous or other systematic and continuous presence in this jurisdiction for the practice of law. y 5.5(c)(4)--> arise out of or are reasonably related to, but can only do so temporarily. y How long is temporarily? States construe this in different ways. y 5.5(d) a lawyer admitted to a US jurisdiction and not suspended from practice may provide legal ser

y Remember Cali lawyer on fed inc. tax matters who puts ads on television. It's possible that this

y

lawyer is allowed through irs rules or case law to provide this service in this jurisdiction where the lawyer isn't licensed. 8.5ay If Cali lawyer is violating laws of NC with these tv commercials, Cali lawyer might also be disciplined by California. y A lawyer not admitted is subject to discipline« y An attempt by states to have some ability to deal with lawyers who are attempting to retain clients in jurisdiction even though they're not licensed in jurisdiction. ------------Murgatroyd; lawyers practice in Cali. Specialized in airplane disasters. Sent letters to relatives of victims in Indiana. The Indiana SC said you can't do this b/c it violates Ind. rules and didn't put 'advertising material' on outside of envelope and didn't file solicitation with state bar. But Ind. didn't have the second sentence in 8.5a. In all cases where they acquired clients they always associated themselves with local lawyers. But they only did that after they had recruited client. Ind.Sc. said these lawyers held themselves out to practice law and thus we have authority to you even though we have never adopted the equivalent in the second sentence of 8.5a

 Leis v. Flynt y Pro Hac Vice - an out-of-state lawyer who has been granted special permission to participate in a particular case, even though the lawyer is not licensed to practice in the state where the case is being tried. y FACTS: Larry Flynt was being sued due to his magazine and wanted to have NY attorney¶s defend him in Ohio. The attorney¶s were not licensed to practice in Ohio and requested admission pro hac vice. Ohio had no governing law or rule regarding pro hac vice admission. The trial court denied the request and the State Supreme Court denied mandamus. Flynt sued in federal court (which affirmed the ruling) and was granted cert by the SC of the US. y RULE: Absent a governing rule or statute, a summary denial of a request to appear pro hac vice is not unconstitutional. y ISSUE: Is there a property interest in practicing law in a state where they are not licensed? 1. Supreme Court says that property comes from state and federal law or a mutually understanding 2. The United States Supreme Court says the lawyers do not have a right to a hearing to see if they can represent Flynn. How can you argue for using lawyers Pro Hac Vice? y 6th Amendment - Right of the D to have assistance of counsel. y 1st Amendment Free Speech ± Lawyers representing unpopular clients. y 14th Amendment ± D¶s right to enter into a K. y Right to Interstate travel y Full faith and credit clause y 14th Am DP guarantees which is applicable to states, and this is a state-level action. y Due Process Guarantee: y Procedural Due Process y Substantive Due Process
y

DPC says states shall not deprive us of life, liberty or property w/o due process of law y Question here is whether there was a Property-interest that the two attorneys have in practicing law. If there is a property interest found here, then the state is obligated to follow procedural due process (meaning state must give person(s) a hearing and the right to produce evidence, must be fair, and must be proceeded by reasonable notice,) and if there is a violation of these attorney's property rights, they must be given this hearing.

y

y

y

y

If that were to happen, the state cannot exclude them is that these two attorneys had represented Mr. Flynt in the past (they were excellent attorneys). y Same court admitted them on earlier occasions Where does this property interest come from? y USSC says that there is no property interest. y Property interest can be found in administrative agency, common law rule, statute, but whatever it is, it must be explicit, not implicit. y That's the majority opinion! Flynt's con. Rights to effective assistance of counsel. Notice in this case that it is the attorneys rights, not mr. flynts rights, that are being considered. Pg 566 w/note. y To have the best lawyer might require getting an out-of-state lawyer. The 1st Am. Case law says that 1st am is to protect against gov't suppression of new ideas. And those ideas are rightfully received and disseminated (with limitations). Attorneys who represent client in 1st am. Case may be the best b/c of their skill to communicate these ideas. Community has a right to hear the ideas advanced, and the best lawyers to do this are not licensed in the state. y So first am. Could very well play role in pro hac vice admissions in the furture. Also, 14th Amendment of liberty guarantee, a liberty interest, in contracting with those we choose and so forth. Does this now allow the attorneys a hearing as to why they would not be allowed pro hac vice status? IS there a liberty interest being infringed? ( a parallel argument could be that he was going to contract with a lawyer and we have a right to enter into lawful contracts. Flynt proposed was not illegal, was not unlawful, just wanted to hire them to represent. y What about the right to interstate travel? SC hasn't pinpointed that right in the constitution. y Full-faith and Credit clause of Art. IV Sec 1. as well. When you are sworn in as attorneys, there's a judicial proceeding. These are the const. provisions Barnett could identify that could be used to overrule Leis v. Flynt.
y

y

If an out-of-state lawyer serves a client in a jurisdiction in which he is not admitted, he may be guilty of the unauthorized practice of law or UPLA as well as those lawyers who are helping him (Rule 5.5(a))

y Court: 5-4 opinion without oral argument: F loses- there is no due process right deserving of protection y No state created property right to appear PHV y No federal right to appear y No right denied, so no right to a hearing on denial y Dissent: o Importance of lawyers being able to practice o OH courts had routinely admitted lawyers PHV, so there was an implicit promise to do so that got taken away. o Due Process clause protects the interest of the bar ³in discharging its responsibility for the fair administration of justice in our adversary system.´ y As a matter of practice, trial judges routinely permit lawyers to appear PHV, sometimes requiring association with local counsel in case there is an emergency hearing or something that requires a quick appearance. y Cases have consistently held (though not unanimously) the right of a client to hire an out of state lawyer including under th the 6 Amendment. y What about lawyers (the great majority) who are not involved in dispute resolution? y PHV is not available to transactional lawyers. o There is no limitation on the scope of work as there is in litigation, where the pleadings define the work allowed. o Yet (a) increased specialization, (b) the uniformity of much state law (and all federal and international law), (c) cost and efficiency, (d) client preferences, and (e) the availability of the law anywhere from anywhere all argue in favor of greater flexibility. y Alternatives:

o National bar admission o One state license is enough: have license can travel o Carving out ³local law exceptions´ to national practice? y Still the problem of control (if you¶re not admitted, you can¶t be disciplined) ---------------------------------

y

y

If an out-of-state lawyer serves a client in a jurisdiction in which he is not admitted, he may be guilty of the unauthorized practice of law or UPLA as well as those lawyers who are helping him (Rule 5.5(a)) What Constitutes the ³Practice of Law´ in a Jurisdiction? j Purpose of Regulations on Lawyers Being Licensed to Practice in a State: Guarantee the competency of the attorney¶s

 Session 3.1: Services Other Than Litigation
 ³Local Office, National Practice´  Problem on 572->making calls, e-mails, website, but can provide these services on a temporary basis y 5.5c: lawyer not admitted to practice in jurisdiction x can provide temporary legal services in x if those services are or in relation to a proceeding or tribunal«. y Lawyer here is drafting employment contracts, y Does he fall under 5.5c2? y Lily is admitted in DC, but has a national practice negotiating employment agreements for high priced executives. y Nearly all her business is outside DC. y Negotiations with employers and meetings with clients can occur anywhere, including the client¶s home state, the employer¶s main office, an airport hotel. y The agreements usually provide that the employer¶s home state law governs y Terms and advice can include salary, pension, profit sharing, stock options, tax issues, benefits, job title and responsibility. y Questions about L¶s work:  On whose law is she advising?  Where is she doing the work?  Can she require her client and the prospective employer to come to DC where she is admitted? o This would probably ruin her practice.  Can she do all the work from DC ³virtually´ without physically entering any state? o It is unlikely that people will challenge her fee in a fee dispute which is where this applies the most. o What about specifying that DC law will apply to all the agreements?  This is not as certain as working in DC, but this won¶t work for her clients. o Can she just not go into any one state too often?  But we don¶t know how often too often is. o If she isn¶t in any one state too long, she probably won¶t be in too long to violate a temporary test.  Birbrower, Montalbano, Condon & Frank P.C. v. Superior Court  RULE: Advising a client and negotiating a settlement agreement in CA without a license constitutes the unauthorized practice of law and no fee may be collected to the extent that the fee was for those services.  CA statute states: ³No person shall practice law in CA unless the person is an active member of the state bar.´  Issues: y Did Birbrower Practice law?

 
y y y

j Majority: Litigation or drafting of legal instruments is the practice of law. j Dissent: Any activity needing a ³trained legal mind´ constitutes the practice of law.  The Court says that a trained legal mind is one that has an understanding of law, is trained in legal research, etc.  The Model Rules don¶t really clarify what a trained legal mind is.  Rule 1.1 Comment 2: It¶s the ability to identify and research legal issues y Was Birbrower practicing law in CA? j TEST: The Court uses a 2 Part Test to determine if Birbrower was practicing law in CA  What is the Nature and Extent of Personal Contacts of the Non-Licensed Lawyer in CA?  Is the service/advice provided involving CA law?  There is no difference between oral advice and written advice y Rule 8.5 (a) Lawyer is subject to discipline in state where they are licensed to practice even if activity occurs in another state. Question was, could the firm recover under the fee agreement. Defense is that they cannot b/c they weren't licensed to practice in the Cali. y Rule 5.5: Was birbrower 'practicing law'? When it was trying to institute arbitration proceedings Maj: 'doing or performing services y If you're giving legal advice, you're practicing law Dissent: y Says you must have an activity that requires a trained legal mind. y What is a trained legal mind? 1. The ability to link legal principles from various principles of the law. 2. Understanding of the philosophy of law.
y

PG 208 Supp: EC3-5. "neither necessary or desirable to adhere to a single definition of practicing law«..calls for the professional judgment of the lawyer«.."

y

3. The ability to identify all the legal issues and facts. a. MR 1.1 comment 2. -> pg 11 supp. 4. Lawyer has the ability to research legal issues and be able to find al areas of facts. 5. Ability to recognize ambiguities in legal doctrines, statutes, decisions. 6. Ability to apply existing legal doctrine, concepts, and principles of law of decisions to novel situation. Were they doing it in Cali? 1. If they were in NY, then no problem. y Practicing law in california y And dealing w/ california law. y Statute is ambiguous, says you don't have to set foot in california. y Mere fortiutious contacts will not be considered as 'in'.

y

UPL will result in the loss of fees for work done in the jurisdiction in which the attorney is not licensed to practice. y A lawyer or law firm may be virtually present in a jurisdiction through e-mail, telephone, or fax machine. y Practicing federal law in the state is no considered practicing law in the state. (separate) 2 Element tests. Rule 5.5 y A basis for disciplinary action if you violate any of the provisions for these rules.

a. A lwyer shalll not practice law in the jurisdiction«or assist another in doing so«. b. Did the birbrower lawyers violate b? (they did violate A).
y 

y

Did they have a systematic and continuous presence?  Was 5.5b.1 violated? Rule 5.b.2  Did they violate this?  Birb said they aren't barred in Cali, but Defendants deny. y They didn't hold themselves out as licensed in california. Rule 5.c -no test for 'temporary'. When does something become other than temporary? y Take note of this!!!

y Firm: y NY law firm represents closely held company in NY y NY client asks firm to represent a related company (ESQ.) on the west coast (common ownership) in an impending arbitration against Tandem. y Partners make three brief trips to the coast to interview expert, negotiate settlement with opponent. y ESQ- Tandem dispute settles y ESQ sues for malpractice; sues firm for fee y ESQ claims firm lawyers engaged in UPL ³in´ CA and can¶t get paid y Firm says ESQ knew lawyers were NY lawyers y Court: y Three trips to state plus the following establish ³extensive´ practice of law ³in´ CA: o Contract with Tandem provided for CA law to apply o CA designated as site for arbitration o Client was CA company o CA law controls firm¶s fee agreement y NY license inadequate to protect CA residents bc ³other states¶ laws may differ substantially´ y ESQ¶s knowledge that Birbrower lawyers admitted only in NY is irrelevant. This is a public policy matter. y ³Mere fortuitous or attenuated contacts will not sustain a finding´ of UPL ³in CA.´ y Representation in connection with pending arbitration is the practice of law. o Dissent: No it¶s not. o Actually, what is ³the practice of law´? o At least pay us for work we did in NY (even on CA law?) o Court says firm can get paid on a quantum meruit basis for some services performed while physically ³outside CA´ that weren¶t virtually in CA. o But ³one may practice law in the state«though not physically present here by advising a CA client on CA law in connection with a CA legal dispute by telephone, fax, computer or other modern technological means.´  Does this make sense? y Why Birbrower was bad for CA and the fallout o If you are CA, you worry that other states will do the same thing to you. o If you negotiate and add an arbitration clause, lawyers negotiating against CA lawyers will not agree to CA or CA venue for arbitration bc they don¶t want to be caught in a Birbrower situation. o CA will lose arbitral business which is a moneymaker.

o y The ABA takes up the issues o When should an out of state lawyer be allowed to practice law ³in´ another state? o How can the ³host´ state protect its citizens against unscrupulous lawyers? o What should be the ³home´ state¶s responsibility if one of its lawyers violates a rule in the ³host´ state? o Should we have a special (³house-counsel´) rule for employed lawyers who have one client and are moved between offices of the client?
Model Rule 5.5 (a) A lawyer shall not practice law in a jurisdiction in violation of the regulation of the legal profession in that jurisdiction, or assist another in doing so. (b) A lawyer who is not admitted to practice in this jurisdiction shall not: (1) except as authorized by these Rules or other law, establish an office or other systematic and continuous presence in this jurisdiction for the practice of law; or (2) hold out to the public or otherwise represent that the lawyer is admitted to practice law in this jurisdiction (c) A lawyer admitted in another US jurisdiction, and not disbarred or suspended from practice in any jurisdiction, may provide legal services on a temporary basis in this jurisdiction that: (1) are undertaken in association with a lawyer who is admitted to practice in this jurisdiction and who actively participates in the matter; (2) are in or reasonably related to a pending or potential proceeding before a tribunal in this or another jurisdiction, if the lawyer, or a person the lawyer is assisting, is authorized by law or order to appear in such proceeding or reasonably expects to be so authorized; (3) are in or reasonably related to a pending or potential arbitration, mediation, or other alternative dispute resolution proceeding in this or another jurisdiction, if the services arise out of or are reasonably related to the lawyer¶s practice in a jurisdiction in which the lawyer is admitted to practice and are not services for which the forum requires pro hac vice admission; or (4) are not within paragraphs (c)(2) or (c)(3) and arise out of or are reasonably related to the lawyer¶s practice in a jurisdiction in which the lawyer is admitted to practice. (d) A lawyer admitted in another US jurisdiction, and not disbarred or suspended from practice in any jurisdiction, may provide legal services in this jurisdiction that: (1) are provided to the lawyer¶s employer or its organizational affiliates and are not services for which the forum requires pro hac vice admission; or (2) are services that the lawyer is authorized to provide by federal law or other law of this jurisdiction. Model Rule 5.5 comment 14 A variety of factors evidence such a relationship. (a) The lawyer¶s client may have been previously represented by the lawyer, or (b) may be resident in or have substantial contacts with the jurisdiction in which the lawyer is admitted. (c) The matter, although involving other jurisdictions, may have a significant connection with that jurisdiction. (d) In other cases, significant aspects of the lawyer¶s work might be conducted in that jurisdiction or a significant aspect of the matter may involve the law of that jurisdiction. (e) The necessary relationship might arise when the client¶s activities or the legal issues involve multiple jurisdictions, such as when the officers of a multinational corporation survey potential business sites and seek the services of their lawyer in assessing the relative merits of each. (f) In addition, the services may draw on the lawyer¶s recognized expertise developed through the regular practice of law on behalf of clients in matters involving a particular body of federal, nationally-uniform foreign, or international law. Model Rule 8.5 y The host state has two interests o If the temporary lawyer misbehaves, the state has to have jurisdiction to discipline.  8.5 gives long arm disciplinary authority o The state has an interest in applying its own rules  8.5 is a choice of rule rule. States can send their disciplinary recommendations to the lawyer¶s licensing state and that state should follow it. (a) Disciplinary Authority. A lawyer admitted to practice in this jurisdiction is subject to the disciplinary authority of this jurisdiction, regardless of where the lawyer¶s conduct occurs. A lawyer not admitted in this jurisdiction is also subject to the disciplinary authority of this jurisdiction if the lawyer provides or offers to provide any legal services in this jurisdiction. A lawyer may be subject to the disciplinary authority of both this jurisdiction and another jurisdiction for the same conduct. (b) Choice of Law. In any exercise of the disciplinary authority of this jurisdiction, the rules of professional conduct to be applied shall be as follows:

(1) for conduct in connection with a matter pending before a tribunal, the rules of the jurisdiction in which the tribunal sits, unless the rules of the tribunal provide otherwise; and (2) for any other conduct, the rules of the jurisdiction in which the lawyer¶s conduct occurred, or, if the predominant effect of the conduct is in a different jurisdiction, the rules of that jurisdiction shall be applied to the conduct. A lawyer shall not be subject to discipline if the lawyer¶s conduct conforms to the rules of a jurisdiction in which the lawyer reasonably believes the predominant effect of the lawyer¶s conduct will occur.

-

Women Serving on Juries y ISSUE: Is it constitutional for a statute to state women don¶t have to serve on juries? 1. Hoyt v. FL ± yes. a. State said women did not have to serve on juries. b. USSC got the case under the 14th Am and concluded that there was no constitutional violation. 2. Taylor v. Indiana ± no. a. USSC said there was a violation under the 6th Am. 3. A state statute saying women don¶t have to serve on juries is constitutional under the 14th Am. but is unconstitutional under the 6th Am.

 El Gemayel v. Seaman  FACTS: P is a lawyer, who is admitted to practice in Lebanon, but not the U.S. He lives in the US and was contacted by a NY woman (D) because her daughter had been kidnapped by her ex-husband and was found in Lebanon. The P negotiated a K with the D and went over to Lebanon and got P¶s daughter back. The P¶s only contact with NY was his phone calls with D and a trip to D¶s house to return luggage that the D had forgotten. P is suing because D refused to pay his bill. P argues that he was not licensed to practice law in NY and therefore the bill/K is void.  Did the P¶s conduct constitute the unlawful practice of law in NY? No. It was too attenuated to be considered practicing in NY. y y

Court says practicing Lebanese law is practicing law. Court rationalized that in our global world a per se rule banning any contact between NY and a lawyer that is not licensed to practice in NY would be too restrictive in our global marketplace. y Court concludes the contacts were 'incidental and innocuous'. j Did it have all of the facts? What other facts could be pertinent? court does not decide whether he's illegally practicing law in washdc or mass. HYPO: P meets D at JFK in NY to travel together to Lebanon. Is this the practice of law in NY?  Birbrower would consider this physical presence in NY in equating this decision but the lawyer does not have to come into state to be found practicing law in that state. 

 Session 3.2: Marketing Legal Services/Free Speech
Marketing Legal Services ISSUE: Is this protected by the 1st Amendment? Lawyer advertising is commercial speech and, as such, is protected by the 1st Amendment.  Bates v. State Bar (USSC)  Arizona restrained print ads by lawyers  RULE: The 1st Amendment Free Speech clause protects truthful advertisement concerning the availability and terms of routine legal services.  RULES: y Rule 7.1 ± A lawyer should not make a false statement about their services or the lawyer themselves.

j Can apply in 1-on-1 conversations j Can be false/misleading by omission/commission y Rule 7.2 (a) Allows a Lawyer to Advertise j A lawyer could be disciplined if someone he hired made false or misleading statements.  Ohralik v. Ohio State Bar. Assn.  RULE: A state may constitutionally discipline a lawyer for soliciting clients in person, for pecuniary gain, under circumstances likely to pose dangers that the state has a right to prevent.  FACTS: Attorney Ohralik (D), upon learning of an auto accident involving an 18 year old woman, twice visited her in the hospital, whereupon she orally agreed to allow him to represent her. She later sought to renege on the agreement. He also solicited as a client the woman¶s passenger. The Ohio State Bar (P) instituted disciplinary rules barring in-person solicitation of clients. D was indefinitely suspended. y Bates was an attorney who advertised in paper. Issue is on print advertising y "routine legal services" and listed prices for each of those services. y In the course of the discussion the court justifies its position by saying that bankers are allowed to advertise and that¶s no problem.
y

Tangible things (vs intangible things) Easier to regulate banks Everything was truthful. That kind of truthful advertising has benefits.



 

    

Rationale behind RULE: A state has a strong interest in preventing the perceived harms of atty. solicitation, these being assertion of fraudulent claims, debasement of the legal profession, and potential harm to the client by overreaching, overcharging, under-representation and misrepresentation. The potential for coercion of the potential client is particularly strong in face to face solicitation, as the atty., who is trained in persuasion, holds a great advantage over the client, who is at best unfamiliar with the law and may be in a particularly vulnerable situation due to whatever misfortune it was that led the atty. to seek him out. y Ct. says that the state is regulating conduct and not speech but there is a speech component j When speech is a part of conduct some constitutional protections may exist y The standard seems to be that: j The state must have a strong interest See Rationale. j The state must act reasonably In person solicitation is different from print ads. States have a strong interest in regulating lawyers ads. NO injury is necessary the rules prohibiting solicitation are prophylactic measures whose objective is the prevention of harm before it occurs. « the potential for overreaching is significantly greater whena lawyer, a professional trained in the art of persuasion, personally solicits an unsophisticated, injured, or distressed lay person. y Unlike the advertising in Bates, in-person solicitation is not visible or otherwise open to public scrutiny. Often when there is no witness other than the lawyer and the lay person whom he has solicited, rendering it difficult or impossible to obtain reliable proof of what actually took place RULE 7.3 Ban on In Person Solicitation of those Known to Need Legal Services y There are some exceptions in the comments section. An Omission CAN be sufficient to create a material Rule 7.1(see comment 2) 7.2 & Comment 5 Lawyer cant hire ad firm and say do what you want. If the ad firm violates the rules, the lawyer can be held accountable. State must have a reasonable basis in believing has a strong interest and has substantially advanced this interest.

y y y y

y

Lawyer cannot hire someone to go to crash site, go to hospital. AGENTS can very well violate the rule by extension. Dignified Advertising y Can a state prohibit advertising that it says is undignified? y For instance, NJ has a rule 7.2a which says the lawyer may advertise«all advertisements shall be 'predominantly informational' and no using techniques that depend on absurdity«.including that contain extreme portrayal« Is the advert materially misleading? Failing to include a disclaimer? Must an advertisement contain statements that submit one's experience or criteria that should be used in choosing a lawyer. PG 203 of MC. Rule 7.3 A lawyer cannot in person solicit professional employment where there is a significant economic incentive for doing so. Unless the person contact is a lawyer or has a« 7.3a. y 7.3b You shall not solicit professional employment even if committed by paragraph a) if the person does not want to be solicited or solicitation involves coercion and duress. y Ohralik would be charged under 7.3 y Commment 1.Suggests that 7.3 applies only when the lawyer knows that the person or persons being solicited is in need of legal services. If the lawyer is aware, then this rule is aware y The implication is if a lawyer does not know. Supreme court says accountants are different than lawyers, accountants deal with objective material. And people go to the accountants¶ office«.The economy depends on the free flow of information.



No duress, influence, undue influence, and the person doing the soliciting who unlike a lawyer is not trained in the techniques of persuasion y Under Rule 7. 3, a lawyer can solicit another lawyer Hypo: a lawyer who specializes in appellate criminal practice contacts a criminal defense trial attorney, saying that he's the best one for appellate work. Is there a problem with that? y Yes perhaps. B/c on appeal, one of the basis that may be used by the appellate attorney, might be, that he use ineffective assistance of counsel at the trial level. y Appellate attorney might have the incentive to pull the punches to do this. y Look at Rule 1.7(a)(2) y There's a conflict of interest here. b/c the appellate attorney has a duty to be loyal to his client Edenfield v. Fane y FACTS: Fane was an accountant who wanted to solicite business from in person office stop ins. y Is it constitutional to ban an out of state accountant from soliciting business in another state through in person office stop-ins? Yes. The Court found that accountants and lawyers are different and that Florida¶s ban on Fane¶s tactics was unconstitutional. y The Fane case introduces some ambiguity into what the state can do. y Questions: y Does Fane constitutionally protect you from your state bar? y Is live telephone contact speech or conduct? y Speech would afford more protection and it would heighten the states burden. y The Fane Case introduces some ambiguity in what a state can do.
y

 Session 4.1 (absent)
 Zauderer v. Office of Disciplinary Counsel ± Targeted Advertisements/Pictures Case  FACTS: Lawyer placed a newspaper advertisement targeting women who had used the Dalkon Shield. The ad informed the reader that the device had been found to cause various physical problems and that D¶s office was handling a number of cases involving the Shield.

HOLDING: A lawyer cannot be disciplined simply for placing an ad that concerned a specific legal problem and that was designed to lure a narrow group of potential clients; there can be discipline if there are misleading statements in the ad. y Commercial speech that is neither false nor deceptive may be restricted only in the service of a substantial government interest and only through means that directly advance that interest.  RULE: A state may not prohibit legal service advertisements targeting a particular segment of the public. (Book).  RULE: Commercial illustrations are entitled to the 1st Amendment protections afforded verbal commercial speech so long as they are accurate and non-deceptive. Illustrations will not be protected if they are likely to deceive, mislead or confuse the reader. (Comp.)  RATIONALE: Government has an interest in prohibiting advertisement that is false, deceptive or misleading or that concerns unlawful activities. However, when these interests are not implicated, the 1st Amendment protects commercial speech. y SC found that OH¶s blanket ban was unconstitutional and that there reasoning behind the ban was unpersuasive.  Was the Ad in poor taste and therefore should it be banned? The Court said that even if it was in poor taste, it did not ³invade the privacy´ of those that read it and is thus allowed. y The Court is vague on whether an ad in poor taste can be sanctioned, how to quantify bad taste, etc. HYPO: Can a lawyer promise to give dinner, gifts, etc. for a person coming in for a pre-trial meeting? Rule 8.1 (e) seems to say a lawyer cannot do this.  Shapero v. Kentucky Bar Assn. Targeted Mail towards those with legal problems  FACTS: P, a Kentucky attorney, applied to the Attorneys Advertising Commission of the KY State Bar for approval of an advertisement he wished to circulate. The advertisement would be sent to those individuals who, according to public records, were facing imminent foreclosure. The ad apprised the recipients that avenues for forestalling foreclosure existed and recommended that the addressee cal P¶s office for a free consultation about their rights in this matter. While not finding the advertisement false or misleading, the D found it to be in conflict with a KY Rule and forbade it.  RULE: A state may not prohibit the mailing of advertisements to a target audience believed to be in need of particular legal services. y Targeted Mail and Advertisements are treated the same both lack the coercive impact and the reader can easily discard.  RATIONALE: The 1st Amendment protects from state prohibition advertising which is neither false nor misleading. The Court found that the policy reasons for prohibiting in person solicitation (coercion, lack of evidence) were not present in this direct mail advertising.  The ABA Response to Shapero: Rule 7.3 was rewritten to permit targeted direct mail to potential clients. However, where the communication is aimed at a person ³known to be in need of legal services in a particular matter´, the words ³Advertising Material´ must appear on the outside of the envelope, if any, and a the beginning and ending f any recorded or electronic communication unless the recipient is a lawyer , a member of a lawyers family or a someone with whom the lawyer has a prior professional relationship.  What to Take from Zauderer, Ohralik and Shapero: Outright face-to-face solicitation can be categorically banned. An advertisement, even one to a target audience and containing illustrations, may be permitted. However, it must not be misleading and must disclose all or most information necessary for a would-be client to make up his mind about using the lawyer¶s services. y  Zauderer v. Office of Disciplinary Counsel (Part IV)  Commercial illustrations are entitled to the first amendment protections afforded verbal commercial speech: restrictions on the use of visual media of expression in advertising must survive scrutiny under the Central Hudson Test. (the gov¶t may bay forms of communication more likely to deceive



the public than to inform it, but there can be no constitutional objection to the suppression of commercial messages that do not accurately inform the public about lawful activity If the communication is neither misleading nor related to unlawful activity, the government¶s power is more circumscribed. The state must thus assert a substantial interest to be achieved by the restrictions on commercial speech). y State¶s law says that the purpose of the restriction on the use of illustrations is to ensure that attorneys advertise µin a dignified manner¶. state undoubtedly has a substantial interest in ensuring that it¶s attorneys behave with dignity and decorum in the courtroom. We are unsure that the state¶s desire that attorneys maintain their dignity in their communications with the public as an interest substantial enough to justify the abridgement of their first amendment rights. Even if that were the case, we are unpersuaded that undignified behavior would tend to recur so often as to warrant a prophylactic rule. y Appellant may not be disciplined for his use of an accurate and nondescriptive illustration.--> indeed, because it is probably rare that decisions regarding consumption of legal services are based on a consumer¶s assumptions about qualities of the product that can be represented visually, illustrations in a lawyer¶s advertisements will probably be less likely to lend themselves to material misrepresentations than illustrations in other forms of advertising. y Conclusion: blanket bans on price advertising by attorneys and rules preventing attorneys from using nondeceptive terminology to describe their fields of practice are impermissible j CT: ad not deceptive or false and were entirely accurate. requires a state to overcome a burden of establishing that prohibiting use of such statements to solicit or obtain legal business directly advances a substantial government interest. ODC Argument: that it is similar to Ohralik (which ct says not a chance«different concerns) ->also argues that the ad µencourages others to file lawsuits¶ (Ct: not quite, again) CT: state posits argument that there are some circumstances that a prophylactic rule is the least restrictive possible means of achieving a substantial government interest«.but the state has not presented a convincing case for this. (state attempts to distinguish between deceptive and non-deceptive legal ads) -State argument proceeds from the premise that it is difficult for unassuming consumers to distinguish between legal truth and deception (ct: Dalkon Shield statement was researchable and accurate as is) O¶connor¶s Dissent: two persuasive arguments to restrict unsolicited legal advice. 1. greater likelihood of deception and confusion in marketing professional services 2. attorney¶s personal interests in obtaining business may color the advice offered in soliciting clients

 Session 4.2
 Florida Bar v. Went for It, Inc. y Procedural Posture: Appeal from judgment which affirmed district court's grant of summary judgment to respondents, an attorney referral service and an attorney, in respondents' action alleging the petitioner state Bar's rules violated the 1st and 14th Amendment of the U.S. Const. y Facts: Plaintiff, an owner of a lawyer referral service, filed an action for declaratory and injunctive relief on grounds of a violation of the 1st and 14th Amendments based on Florida Bar Rules creating what is essentially a 30-day blackout period for lawyers wishing to contact through directed mail those who have been injured in an accident. FB was concerned with protecting victims of disasters and relatives when they were emotionally vulnerable. That is not the case here, Maryland was not trying to protect those who were emotionally vulnerable«in conjunction with this, that time is of the essence when someone has been charged with a crime. In civil suit, the potential plaintiff had three years with which to decide to pursue a claim.

Criminal D needs immediate action. Sixth Amendment says you have a right to counsel when charged with a crime--> no such provision for civil suits. Fourth Circuit Said that privacy is not an issue in the Maryland case b/c privacy of D has already been breached by the arrest and booking. 4th Cir. Adds some caveats, a ban on personal solicitation would probably be sustained, this case did not involved phone calls and so the Fickert case with letters targeted to the accused «.-y y y Issue: Whether the Florida Bar rules violate the 1st and 14th Amendment Rule: The government may regulate commercial speech Commercial speech that falls into neither of those categories may be regulated if the government satisfies a test consisting of three related prongs: (1) the government must assert a substantial interest in support of its regulation, (2) the government must demonstrate that the restriction on commercial speech directly and materially advances that interest, and (3) the regulation must be "narrowly drawn." (Hudson) Holding: No. The government has surpassed intermediate scrutiny on the restriction of commercial speech by presenting a strong showing to satisfy each of the three Hudson prongs. Analysis: Prong 1: State¶s substantial interest in support of regulation: protect reputation from acts committed by lawyers that are beneath the public decency, in addition to protecting the privacy and tranquility of personal injury victims against intrusive and unsolicited acts by lawyers. Dissent; Privacy interest is not applicable, citing Shapiro¶s distinction between inperson solicitation and direct-mail solicitations. The focus is on the mode that poses danger, and the mode of mail is not dangerous to the public. Also cites Zauderer striking down a ban on advertising even if some members find it objectionable does not warrant suppression of it. Prong 2: Government demonstrated that the harm is real and the regulation advances this interest of protecting against it being committed: cited the Fla. Bar study which found that Floridians viewed direct mail solicitation in the immediate wake of accidents are intrusions into their privacy which thus reflects poorly on the profession. Cites specific examples of comments made in inquiry. Dissent: argues that the sample size or selection procedures are inadequate. Majority says that it does not read case law to require empirical evidence data to be introduced with a surfeit of background information. Also says it is µcommon sense¶ that this reaction happens. Prong 3: The government regulation is narrowly drawn because the interest as against the means used to protect them is reasonably fit. By the simple fact that the scope of the regulation pertains only to injuries from accidents with which harm physically and emotionally, and this is reasonably tailored to its stated objective of eliminating targeted mailings whose type and timing are a source of distress, and also of which causes a loss of respect for the legal profession. Dissent: Rule is overinclusive, and also may prevent citizens from learning about legal options. Maj. Says that the regulation, instead of being based on severity of injury, reasonably protects all victims for a brief period of time. Also articulates the variety of other avenues people learn about legal recourse (billboards)

y y y

y

y

y

y

y

RULE: Florida Bar rule prohibiting personal injury lawyers from sending targeted direct mail solicitations to victims and their relatives for 30 days following an accident or a disaster is constitutional.

y

y
y y

CONSTITUTIONALITY TEST ± (From Hudson ± Intermediate Scrutiny) Government may regulate if they satisfy: 1. The government must assert a SUBSTANTIAL INTEREST in support of the regulation. o Substantial Interest here is to protect the privacy ad tranquility of personal injury victims and to protect the reputation of the legal profession. 2. The government must demonstrate that the restriction no commercial speech DIRECTLY and MATERIALLY advances that INTEREST. o State must provide evidence. o P showed statistical findings evidencing the advancement of this interest. 3. The regulation must be NARROWLY DRAWN. o Court found that FL Bar¶s Rule was narrowly drawn and that there weren¶t any better alternatives. o Doesn¶t have to be the least restrictive means. The Court seems really concerned about the public view of the legal profession being harmed.

What is the Standard of Constitutionality What are the Tests that the court adopts in assessing the constitutionality 1. Must assert a substantial interest: which interests were asserted by FLA bar? People after disaster tend to be vulnerable and state wanted to protect their privacy.
y y

Keep in Mind Rule 8.4(d) Engage in conduct that is prejudicial to the administration of justice--> does this mean that anything they do that harms the reputation can be regulated?

2. Test 2 breaks into two subparts 1. Interest must be asserted by government to support its regulation must be real and 2. Regulation must substantially advance that regulation to protect against the harm.
y

Dissent touches upon problems in the statistics y Point is that the legal system is going to use survey data, it ought to use it correctly and thoroughly and not use superficial data, especially under 1st Am. Of Const. --> some of these surveys are tapping into those people who have never received a letter from a lawyer«.should the data be confined to the attitudes and actions would be protected by the bar's ban. Was this limited to english-speaking persons? Who was in sample? -Adults? Was it weighted with number of adults in household? What is the response rate? How many completed the interview? Did the questionaire reverse the questions so that the alternatives would pick up? What was the wording of the questions? Questions in survey that would allow arespondent to express the view that advertising by letter improved the view of the legal question. Where there results unfavorable to the results that they wanted? If so: this potentially is a violation of rule 3.3(a) and note 3. Not knowingly make a false statement. What is knowingly? Was it constructive knowledge? Also 3.3c

3. Third test, regulation vs. ends pursued. Means v. ends. Reasonable and proportion. Notice the dissent which argues in part that what we have here is more than the protection of the privacy of the people who have been hurt, but also of the legal profession. Potentially the denial of access of information.

y y y

Consider that the case could be settled under 30 days. Insurance companies are not banned from going door to door. Ohralik is in consideration. Rule 7.1 and comment 3. y Ads handout y "harvard lawyer" y Is it misleading to not articulate what role he had in drafting the law? y Pape Chandler: got a public reprimand and went to ad lecture to help. Comment 3 to rule 7.1: its' getting at what FLA Bar rule said. Fla SC said that this tv commercial was just like the internet ad conveyed a message of quality of the lawyers services. Court said the pitbull y Must be 'objectively relevant' to the role of an attorney. Ct says that image as Logo would suggest that they get results and is inherently deceptive b/c it cannot be verified.

y

HYPO: Can a lawyer post DUI check points and speed traps on his webpage? y No. Rule 1.2 (d) ± Lawyer shall not counsel a client to engage, or assist a client, in conduct that the lawyer knows is criminal. Rule 8.4 (d) ± Lawyer shall not engage in conduct that is prejudicial to the administration of justice.

 Session 5.1 ± CONCURRENT CONFLICTS OF INTEREST
Types of conflicts: y Concurrent (with other clients, 3rd persons, self-interest) y Successive (with duties to former clients) j Duty to current client is in tension with duty to former clients. Duty to former client is less, but it does exist. There is certainly a duty to maintain continued confidentiality. y Imputed conflicts (a firm is one lawyer (Rule 1.10(a)) j Conflict of one lawyer in a firm becomes the conflict of other lawyers in a firm. y Lateral lawyer conflicts j Screening- a few jurisdictions (but not the model rules) use this to facilitate movement between firms without the lateral lawyers conflict being imputed to the new firm; not all jurisdictions have accepted it. There is some ambiguous case law in NY, but no NY rule. j Former government lawyers: a special rule- encouraging people to go into government service. This gets discouraged if it is difficult to get employment later bc of conflicts as a result of their government service. Most jurisdictions allow screening for government lawyers. y Conflicts in entity representation  -consent? Must be in writing. Key Conflict Provisions in the Model Rules: y 1.7: current client conflicts: the generic rule 

y 1.8: special current client conflict rules y 1.9: former client conflict rules y 1.10: imputation rules y 1.11: rules for former government lawyers y 1.12: rules for judges (and law clerks) y 1.13: rules for entity lawyers What is a current client conflict? Rule 1.7(a): ³A concurrent conflict of interest exists if: (1) the representation of one client will be directly adverse to another client; or (2) there is a significant risk that the representation of one or more clients will be materially limited by the lawyer¶s responsibilities to another client, a former client or a 3rd person or by a personal interest of the lawyer.´ B. Lawyer-Client Conflicts Rule 1.8(a): The rule provides procedural and substantive protection: ³A lawyer shall not enter into a business transaction with a client or knowingly acquire an ownership, possessory, security or other pecuniary interest adverse to a client unless: (1) the transaction and terms on which the lawyer acquires the interest are fair and reasonable to the client and are fully disclosed and transmitted in writing in a manner that can be reasonably understood by the client; (2) the client is advised in writing of the desirability of seeking and is given a reasonable opportunity to seek the advice of independent legal counsel on the transaction; and (3) the client gives informed consent, in a writing signed by the client, to the essential terms of the transaction and the lawyer¶s role in the transaction, including whether the lawyer is representing the client in the transaction.´  Rule 1.7(a) ± A concurrent conflict of interest exists if: (1) the representation of one client will be directly adverse to another client; or (2) there is a significant risk that the representation of one or more clients will be materially limited by the lawyer¶s responsibilities to another client, a former client or a third person or by a personal interest of the lawyer. exception: the lawyer believes he can provide competent and diligent representation to the affected client, Representing Bin Laden: RULE 1.16 ± Declining or Terminating Representation RULE 1.2 ± Scope of Representation (b) representing client does not mean lawyer approves of clients views or activities.  Mark Hansen, Death¶s Advocate  Implied that she left b/c of conflict of interest w/ her boss y Rule 1.11:   In re. Michael Murphy y FACTS: Attorney disciplinary proceeding was brought. The Court held that the attorney who invested in business deal (buying of land) promoted by firm clients and who arranged for other clients to invest in same deal violated rules of professional conduct regarding confidentiality, conflicts of interest, and business dealings with client, warranting one-year suspension from practice of law.

o Murphy was both an investor and an attorney for other investors. o He used inside knowledge, that promoter¶s were having problems monetarily, to guarantee himself a return of 10% annually. This was at the expense of the other investors. y If lawyer abuses trust and loyalty of firm's client for benefit of himself or others, disbarment is generally appropriate sanction. However, in this case the Court found that the D had 25 plus years of being a lawyer with no disciplinary violations ± was mitigating factor. o Litigator not transactional attorney, never billed client, only violated 1.7a, and 1.8a y Murphy violated: o RULE 1.6 (a) Duty of Confidentiality ± he was the attorney to the investors and owed them this duty Violated it by using this information against them. o Conflict of Interest y Was there an attorney-client relationship? YES. There is a presumption that information is circulated through a law firm, and Murphy had a duty to protect against this.  Did he have two clients whose interests were DIRECTLY ADVERSE? Yes the interests of the trust were clearly adverse to those of the promoters. y Duty arose in two ways y Common law duty/principle that an attorney in a firm is presume to know the information possessed by every other attorney. Court cites an ABA publication and does not refer to 1.10. There was a common law proposition that attorneys in a firm are assumed to share the information of every attorney in the firm. y Could separately use 1.10: 1.10a: it does NOT mention rule 1.6 in its text, does not incorporate rule 1.6: it incorporates explicitly, 1.7 and 1.9. y 1.6 says lawyer shall not reveal confidential information. y 1.10 simply does not bring into play rule 1.6. y Rule 1.7, incorporated by reference by 1.10, 1.7 says, there can be a concurrent conflict of interest if there is a significant risk with which a lawyer is materially limited«. y Two issues y Confidential informaiton y Conflict of interest 1.6 vs. 1.8b 1.6a: lawyer you shall not disclose, reveal, confidential client info 1.8b: lawyer, you shall not use client information to the disadvantage of client. Trust v. T&A--> what is the meaning of directly adverse: Mr. Murphy's role as an investor in the project y He argues he didn't violate 1.10, b/c 1.7 and 1/9 are the only rules incorporated by reference into 1.10. He says you cannot discipline under 1.8a, because 1.10a does not mention 1.8a. Court concludes that this rule 1.10 applies to 1.8a to Mr. Murphy saying 'we fail to see the logic'. He puts forth a plausible logical argument and court says 'we don't care'. You still violated it by representing yourself in this transaction as well as the trust. Rule 5.7 plus comment 9 y Looks that Murphy, with trust, was providing a law related service. y Even if he was not strictly acting as a lawyer«he could be under an obligation to follow these rules. y Model rules do not permit a lawyer to advance more than court costs and the expenses of litigation. (Both excludes living and medical expenses no humanitarian exception, except perhaps as a small gift like 200 dollars for basic needs.

Rule 1.8 is designed to prevent clients from selecting a lawyer based on improper factors and avoid conflicts of interest.  ³MAY The Lawyer Be Our Client?´ y Jen is an IP lawyer. Rich Bellow represents an opposing client in a litigation. y Rich¶s firm got sued for malpractice and hires Jen¶s partner Nola to defend them. y Rich was not involved in the malpractice. y Jen: ³Do we have a conflict bc Rich¶s firm is our client while Rich is my adversary?´ y Is either firm representing two clients that are ³directly adverse´? j No, according to Rule 1.7(a)(1). y Does Jen¶s firm (the only one with two clients) have inconsistent responsibilities to them as defined in Rule 1.7(a)(2)?: y there is a significant risk that the representation of one or more clients will be materially limited by the lawyer¶s responsibilities to another client« j NO. The scope of the representation of RB¶s firm (defending malpractice) does not create responsibilities not to create obligations in another case. There is no obligation not to be an opposing firm to a client, but you can¶t be an opposing firm to the other firm itself as a client, that is not a client of the opposing firm. j Focusing on RB¶s firm, the scope of the retainer and nature of the responsibility do not obligate Jen¶s firm to any professional duty not to be adverse to RB¶s firm in its representational capacity. y Does either firm have a conflict between its interests and the interests of its client? j This is client interest v. lawyer interest. j (2) there is a significant risk that the representation of one or more clients will be materially limited by« a personal interest of the lawyer.´ j RB¶s firm may be a big malpractice client. This may be ongoing income for Jen¶s firm. Jen may not want to antagonize RB¶s firm by vigorously litigating the other case she has. The clients should be able to make this decision. j There is risk of materially limiting the lawyer¶s work for the client.   Matter of Neville y Attorney with fiduciary duty must act in the best interests of the client. y Comment 4 to Rule 1.3--termination of relationship«client may sometimes assume that the relationship continues unless the lawyer has given notice of withdrawal. y Rule 1.8A j Neville should have disclosed all terms advantageous to him and those terms which where disadvantageous to Bly.  Client must give informed consent. y y FACTS: Attorney Neville (D) represented Bly (P), a licensed real estate broker, in certain real estate matters. D also purchased options in certain of P¶s property. D, P and a third party then entered into a K, drafted by D, under which P¶s property would go to D in exchange for a promissory note. P created the substantive terms, and D accepted these terms with no negotiation. The AZ Bar charged D with violation of rules governing attorney-client business deals. y RULE: Whenever lawyers knowingly acquire an ownership, possessory, security or other pecuniary interest adverse to a client, the client must be given a reasonable opportunity to seek the advice of independent counsel. Model Rule 1.8 (Specific Rules for Conflict of Interests with Current Clients) asserts this RULE. y RATIONALE: o Model Rule 1.8 is not limited to those situations in which the lawyer is acting as counsel in the very transaction in which his interests are adverse to his client. It applies also to y

transactions in which, although the lawyer is not formally in an attorney-client relationship with the adverse party, it may fairly be said that because of other transactions, an ordinary person would look to the lawyer as protector rather than adversary. o The Courts make it more difficult for lawyers to deal adversely with past and present clients because it is believed that this result conforms to the obligation of the profession and is in the public interest. o Attorney¶s have Fiduciary Duties to Clients. y TEST: Does the Lawyer still have influence over the client? y YES ± Still have a Fiduciary Duty. y NO ± No Fiduciary Duty. o In order to no longer act in clients best interest, lawyer MUST fully disclose and receive consent. y Here Neville failed full disclosure. y A lawyer has an affirmative duty to make client understand and consent. HYPO: Lawyer is a partner at a large law firm and General Counsel for a large company. His job with the company is to find a law firm to help with legal troubles. Is there a problem with RULE 1.8 Conflict of Interest with Current Clients? Can he hire a law firm that he is already partner in?  YES. There will be no conflict of interest if there is not a significant risk that the lawyers representation of the company will be materially limited by the lawyers responsibility to the firm, and he will be allowed to hire his firm. In this case, it doesn¶t seem that the interests of the company and the law firm are adverse, but contrarily that they are in synergy.  With regard to 1.8a.  RJR Hypo: lawyer in a firm, hired onto gen counsel for RJR. Is there a violation of 1.8a.  Does the lawyer have an adverse pecuniary interest by being employed in both places where RJR had hired his firm previously and continues doing business with the firm with which he is employed? y What is the definition of 'business transaction'. --> Does the meaning of 'transaction' imply being hired to an employer>? j Everytime you're hired by a client, it's a business transaction. If it is, then A(1)(2)(3)«.particularly (2), advising client«.  What about "knowingly acquiring' a pecuniary interest adverse to client? y Does lawyer come within that part of 1.8a? j The way 1.8 is worded implies that the adverse pecuniary interest must come into existence AFTER the  "acquire" and "acquires" --> present tense.  He already had partnership in law firm. And RJR knew this. y Time sequence: this case, he's already a partner in the firm so he already had the adverse pecuniary interest adverse to the client Some other Current Client-Lawyer Conflict Rules: y 1.8(d) ³Prior to the conclusion of representation of a client, a lawyer shall not make or negotiate an agreement giving the lawyer literary or media rights to a portrayal or account based in substantial part on information relating to the representation.´ y 1.8(e) ³A lawyer shall not provide financial assistance to a client in connection with pending or contemplated litigation, except that: y a lawyer may advance court costs and expenses of litigation, the repayment of which may be contingent on the outcome of the matter; and y a lawyer representing an indigent client may pay court costs and expenses of litigation on behalf of the client.´ y 1.8(f) ³A lawyer shall not accept compensation for representing a client from one other other than the client unless: y the client gives informed consent;

y there is no interference with the lawyer¶s independence of professional judgment or with the client-lawyer relationship; and y information relating to representation of a client is protected as required by Rule 1.6.´ y 1.8(j) ³A lawyer shall not have sexual relations with a client unless a consensual sexual relationship existed between them when the client-lawyer relationship commenced.´ y 1.8(k) ³While lawyers are associated in a firm, a prohibition in the foregoing paragraphs (a) through (i) that applies to any one of them shall apply to all of them.´ y 1.8(e) is especially important for indigent and near indigent clients. If is injured and needs money, has money. Lawyer is working on contingency. knows needs money and uses procedural rules to delay the trial. asks lawyer for a loan to pay the bills. ABA and NY rules don¶t allow it. The lawyer can advance costs of litigation like testimony, etc., but no other support no matter how small. o The original theory said it gives a lawyer an interest in a matter that might skew her judgment. The lawyer may encourage a settlement to recoup her costs. This is ridiculous bc these loans are likely much smaller than the other costs of the litigation. o Some states allow humanitarian assistance. o 2nd theory says if we allow this, lawyers will compete for choice clients by offering cash. People will choose lawyers based on cash advances instead of on skill.

 Session 5.2
 Gellman v. Hillal: representing clients previously represented by spousal partner  FACTS: Gellman (P) sued Hilal (D) for medical malpractice. P was represented by Bogaty, whose wife, Brody, had previously represented D in another malpractice action whose subject was the same medical procedure challenged by P. D therefore moved to disqualify Bogaty in the current action. D argued that if Brody divulged her knowledge to Bogaty, D would be prejudiced. D also argued that, as Bogaty¶s wife Brody had a financial incentive to aid Bogaty in the prosecution of P¶s suit since any contingent fee he earned would likely benefit the Brody-Bogaty marital household. D also alleged a danger of inadvertent disclosure in the ordinary course of spousal intimacy of daily life in a shared household.  ISSUE: May lawyers who are related as parent, child, sibling or spouse represent direct adversaries if the clients consent after consultation? YES.  RULE: Lawyers who are related as parent, child, sibling or spouse may represent direct adversaries only if the clients consent after consultation. y In the context of attorney-spouses working for opposing law firms, there is no per se rule of disqualification based on marital status.  RATIONALE: In this case, D relied solely on the hearsay, vague and conclusory affidavits of his counsel Thus did not allege facts sufficient for a court to infer that Bogaty is privy to confidences or other strategic information that if revealed might injure D in this action or that even if she had such knowledge, she has or will improperly divulge it to the D¶s detriment. y Can¶t assume that just because they are married they will not break the ethical rules for lawyering though Bogaty and Brody must follow these rules. y Court here redefines marriage as two people living together, but are independent of one another.  Court says re: marriage that there is a significant risk of inadvertent breach of confidence  "marital couple is an association of two individuals with separate emotional and intellectual makeup .it's
two people who are living together y See: canon 9 pg 252 of Ny MC. y 1.7 Comment 11 j -family relationship will interfere w/ loyalty and independent professional judgment j Relation to lawyer (parent, child, sibling, spouse) may not represent unless client is disclosed and has fully consented  How far do blood relationships go?

  y y

"For example including but not limited to" Can be construed very broadly

 ³KAREN Horowitz¶s Dilemma´  which of the model rules are applicable? y What rules could karen violate if she stayed on? j 1.3: loyalty to client's best interest: firm has best interest of client in mind.  Explicit in comments. j Competent representation? 1.1? --> is she providing competent representation if she insists on y
going? Hypo: j Rule 1.13: j Rule 2.1: "to other considerations that may be relevant to the client's situation"  1.8b: not a situaton that where nola should take on the bellow firm in this litigation. j Rule 1.8k; applicable to one, applicable to all.

  Haley v. Boles  FACTS: Haley was appointed to be the attorney for D by the Court. Haley petitions the Court to revoke this appointment due to the conflict of interest (Haley¶s partner¶s spouse is the prosecuting attorney).  The Court found that the Price¶s (his partner) marital relationship creates the appearance of having compromised and limited the D¶s constitutional right to effective assistance of counsel, and for this reason alone Haley¶s petition must be granted. y There would be a public erosion of confidence in the legal system if Haley was allowed to represent D. Here in this case it would the appearance of independence of the trial counsel that is diminished. y There would be an appearance of a conflict of interest financially because either attorney could stand to benefit from this case monetarily and this would look bad in the eyes of the public. y COURT HERE ADOPTS AN APPEARANCE OF IMPROPRIETY STANDARD EVEN THOUGH ITS NOT IN THE RULES.  Court found there is a difference between a paying client and an indigenous client Indigenous clients don¶t get to pick their attorney¶s (No Consent) and therefore need greater protections.  The Court is very much influenced by the possibility that forcing Mr. Hale to defend against his partner¶s spouse will give an appearance of biased/ineffective counsel. y Last sentence of comment 11 to 1.7: disqualification arising from a close family relationship is personal and is ordinarily not imputed to members of firms with whom the lawyers are associated. j Court repeatedly refers to the appearance that something is amiss the appearance of
impropriety .

j  Formal Ethics Opinion 1992/93-12 of The New Hampshire Bar Assn.  ISSUE: Can lawyer X represent a buyer or seller in a RE matter if his wife is the broker or a principal in the brokerage company? y NH says it would be OK so long as they abide by the Full Disclosure/Consent requirement but that it is still questionable as being good practice. y The lawyer has a personal interest in the matter (making $ by deal going through) and it seems that the Conflict of Interest Rules would not allow these actions.

y

An a lawyer personally represent the buyer in a property transaction where wife of lawyer is personally listed or is listed through her office. y Direct financial conflict of interest, but other courts have said to the contrary.  Model Rules pertinent: 1.7, 1.8a y Interest would be 'marriage' and economic relationship as well as emotional relationship. If adverse pecuniary interest is the marriage, did he make this in lieu of clients interest (already married) y Barnett doesn't see how 1.8 is applicable. y Lawyer would already have the adverse interest and he'd have to disclose«but doesn't see how 1.8 is applicable. y Focus against is 'acquire'«.present tense looking forward. Marriage is in the past.  MR. 1.7a(1). Conflict is not between clients, it's between the attorneys interest against the other spouse's interests. y a(2): personal interest: concerned with how much their spouse makes«.can personally benefit. y Concurrent conflict of interest if there is a significant risk of one more more clients will be materially limited by lawyers responsibilities to another client, a former client, or a third person y Lawyer could have an interest in wife doing well and the viability of her real estate office. y Does then lawyer x have responsibility to wife that materially presents an inhibition to his effective representation to the client«..probably
y y y y

y y y  y

IF SELLER 1.78 If BUYER 1.7/1.8 Attorneys w/in firm must strictly comply w/ the conflict waiver provisions of Rule 1.7. If the lawyer/spouse is personally representing the buyer, the stricter waiver provisions of 1.8(a) must also be complied with. Rule 1.10a applies the imputed disqualificaitons doctrine to conflicts of interest under Rule 1.7, but not to 1.8.a Loyalty: 1.6, and 1.8b are also in play

1.10a-->does not represent a significant risk of materially limiting the representation

Ethics committee says on pg 62 that there may be 'an appearance of impropriety"«..although not in our rules, it's a common law doctrine. j



 Session 6.1 ± CLIENT CLIENT CONFLICTS
CRIMINAL CASES Issues of concurrent conflicts between clients in criminal representation arise when a single lawyer represents two or more suspects or defendants. y The big implication in these cases is post-conviction appeals asserting the client was denied effective assistance of counsel due to conflict of interest.
y y The sixth amendment says the accused shall have the effective assistance of counsel. Standard 4-3.5(pg 452Supp)

 Cuyler v. Sullivan

y

y y

FACTS: Sullivan (D) was accused of murder, along with two accomplices. D was tried separately, his trial occurring first. He was represented by two attorney¶s, who also represented the accomplices. His attorneys rested without presenting evidence. D was convicted. He appealed, contending that a conflict of interest existed as to his representation. The conviction was affirmed. He petitioned for habeas corpus. At a hearing, one attorney testified that the decision not to present a case was due to a weak state case. The other testified that he didn¶t wish to reveal the testimony of certain witnesses, in view of the upcoming trial of the accomplices. The district court denied habeas corpus, but the court of appeals reversed, holding that since a possibility of a conflict existed, D had been denied effective counsel. The SC granted review. No objection was made by D to multiple representation at trial. The two other D¶s were acquitted in separate trials. y RULE: The mere potential of a conflict of interest in representation is not sufficient to invalidate a conviction. y In order to demonstrate a violation of 6th Amendment rights, a D must establish that an actual conflict of interest adversely affected his lawyer¶s performance. y RATIONALE: The potential for a conflict of interest exists in every situation involving multiple representation, so to hold that the mere potential of a conflict is sufficient to invalidate a conviction would end multiple representation. However, in many cases multiple representation actually improves the position of the D. In light of this, this Court believes that only an actual conflict, as opposed to a potential conflict, should invalidate a conviction. Prejudice is presumed when a conflict exists, but should not be when one is only possible. The decision of the appeals court is reversed. y What does the 6th Amendment require of the trial court judge? --> y When you have multiple representation in a criminal proceeding, and no objection made to the common representation until after they're convicted; According to holloway, the trial courts ruling is that unless there is a special circumstances,
y

Unless the trial court knows or reasonably should know (constructive knowledge) of the problem of multiple representation, the trial court need not inquire further. Brennan/Marshall: under the 6th Am., the judge has a responsibility to inquire into the conflicts of interest. The trial court must point out to defendants the possibility of the conflicts of interest and understand what they may be getting themselves into.

y

y

y

2nd Const. Issue: ok, no objection at trial but one after trial, what must be proven by the defendant's on the ground of a 6th Am. Violation? y That the defendant must demonstrate actual conflict of interest which the conflict adversely affected the lawyer's performance. y Majority points out that the defendant need not demonstrate that he was hurt, that there was prejudice. y Brennan says that there has to be a significant possibility of conflict. y Marshall says both are too harsh: Marshall says there must be an actual conflict of interest to the case at hand. 3rd Issue: on which party does the burden of proof fall. y Duty of defendant to raise conflict at issue. y Burden of proof is on the defendant. Must prove that there was a conflict of interest. y Brennan, dissenting, says there should be a presumption that his representation in fact suffered. y Marshall: a showing of an actual conflict of interest requires a reversal, automatically. y So no burden of proof: gov't automatically loses.

y y

y

y y y y y y y

y y y

y

Holloway v. Arkansas (US 1978) (This is a notes case) FACTS: Trial Court appointed a public defender to represent three defendants in the same trial. The lawyer repeatedly requested separate counsel, citing conflicts of interest, but the judge refused to consider the request. Holloway RULE: Where a trial court fails to investigate the alleged conflicts of interest that are objected to by counsel; requires automatic reversal without the need to show any harm at all or ay effect on counsel¶s performance. Dibona and Peruto represented Sullivan and two others indicted for 1st degree murder. The lawyers were paid by the two others. Sullivan was tried 1st and convicted. The lawyers did not call the other to testify for Sullivan. The two other were thereafter acquitted. Sullivan sought relief claiming ineffective assistance of counsel (6th Amendment). y To what extent does the amendment draw on conflict doctrine and how does it apply? The holding in Holloway v. Arkansas y Legal aid lawyer representing multiple clients. Lawyer asked for separate counsel and judge didn¶t really inquire. Clients were convicted. The court said when a lawyer brings a conflict claim, the judge has a duty to investigate and take corrective steps if necessary. Why Holloway does not help Sullivan. To show ineffectiveness based on conflict the must prove that y ³an actual conflict of interest adversely affected his lawyer¶s performance.´ Compare the Strickland v. Washington test- other kinds of ineffectiveness were alleged. y The court imposes a higher burden of showing ineffectiveness before an accused can get relief. must show there is reasonable probability that but for unprofessional errors, the result would have been different. y In Cuyler, the proof is lower; all they have to show if an affect on the lawyer¶s performance. He did not have to show actual prejudice- that the case might have turned out differently. The case on remand y Peruto explained his strategy in defending Sullivan that was a textbook example of conflict affecting his performance. y Why Peruto did not call Carchidi- Peruto didn¶t call Carchidi bc of his duty to Carchidi. That¶s the conflict. th y Lawyer said that even with another lawyer, the would have taken the 5 and wouldn¶t have testified anyway. y PA¶s proximate cause argument rd y 3 Circuit grants the writ: y Sullivan had only to show an effect on the lawyer¶s performance. He did not have to show ³actual prejudice´- i.e., that the case might have turned out differently.

Mickens v. Taylor (p. 217) y Mickens was charged with killing Hall. y Hall was then on bail on an unrelated charge. y Saunders had been appointed to represent Hall on that charge. y After Hall died, the same judge who relieved Saunders from representing Hall after Hall¶s death then appointed Saunders days later to represent Mickens for killing Hall. y Mickens was convicted and sentenced to death. y Holloway distinguished (and see Campbell v. Rice p. 218) o Scalia said Holloway doesn¶t apply bc Saunders didn¶t call the conflict to the attention of the judge. Holloway was granted relief bc the lawyer representing him was a in an unrelated case herself. She may not want to antagonize the prosecutor going after her by aggressively going after that office in another case. o But not bringing up the conflict is evidence of the conflict itself. o Compare with Sullivan- did conflict adversely affect his performance?  Cuyler/Sullivan relief was not available. y Sullivan¶s holding questioned in dicta (5-4).

o o

o

Scalia said Cuyler was a current client/client conflict. There is never a benefit from this test where the conflict is former client/current client as in Mickens. It also doesn¶t apply when it is a conflict between the current client and the lawyer. The lower courts have assumed Sullivan applies to all conflicts.

 MURDER AT THE BALLGAME y Shari LaGuardia is murdered at an afternoon ballgame at Reynoso Park. y Suspicion focuses on her estranged husband, Pete, whose threats had led her to get an order of protection. y Husband¶s alibi is weak, but no evidence puts Pete at the ballpark that day. y Circumstantial evidence implicates Potero, who is charged. y He is represented by Hinajosa. y Hinajosa¶s partner, Park, currently represents Pete on theft from vending machine charges and has represented him in past such cases. y Prosecutor Chen moves to disqualify Hinajosa bc her firm¶s representation of Pete prevents her from arguing to a jury that Pete did it, which could create reasonable doubt. y Hinajosa: ³Blaming Pete would be as good as saying µlook how pathetic we are.¶ I have better. Anyway, Potero will waive the conflict.´ y Chen: ³I can¶t rely on the waiver. You can¶t even consider a µPete did it¶ defense. I don¶t want to convict, then face an ineffectiveness claim.´ y Judge thinks it is unlikely he will be reversed if he disqualifies lawyer. j Even if disqualified, Potero would have to show prejudice. j But if Hinajosa is not disqualified and Potero is convicted, the habeas lawyer will have a strong conflict claim and the prosecutor may lose his conviction. j Trial judge may think it is better off to remove her than to let her stay. j Hinajosa can¶t really get informed consent in this case; there¶s lots of evidence of conflict. y Hinajosa: 1.10 comment 2.  United States v. Mett
y

y

When you have a two prong test, both prongs must be met for the sixth amendment. If one is not met, then the 6th amendment was not violated. y Two prongs: 1. Must be an actual conflict of interest 2. Lawyers performance adversely affected Failure to get a benefit should not be considered a conflict of interest
y

y

y

FACTS: Assistant US Attorney Osborne was arrested for drunk driving. He retained Cassiday as counsel in the matter. US Atty. Osborne was prosecuting Center Art Galleries (founded by Mett) and Cassiday was defending them (as local counsel along with four other attorneys). All parties appeared to have waived any possible conflict orally, nothing was put in writing and the district court was not informed. At the urging of the United States Attorney for Hawaii, Cassiday ended his representation of Osborne just before Osborne pled guilty. Later, a jury convicted Center Art (Mett) of numerous counts of mail and wire fraud. In this trial, Cassiday acted as local counsel and helped with jury selection. Mett contends that he had ineffective assistance of counsel due to the conflict of interest. RULE: The purpose of the 6th Amendment right to conflict-free counsel is to ensure that the defendants are not harmed by their counsel¶s conflicts of interest. Its purpose is not to ensure that defendants benefit from those conflicts of interest. RATIONALE: The 6th Amendment entitles criminal defendants to the effective assistance of counsel, which includes a right to conflict-free counsel. In order to prove a violation of this 6th Amendment right a D must pass the test set forth in Cuyler: ³A defendant who

raised no objection to conflict at trial must demonstrate that an actual conflict adversely affected his lawyer¶s performance.´ The Court found that Mett did not pass this test, and asserted that ³We do not believe that being deprived of a potential benefit from an attorney conflict of interest should be counted as an adverse effect in the Cuyler test.´ Contrarily, an adverse effect in the Cuyler sense must be one that significantly worsens counsel¶s representation of the client before the court or in negotiations with the government. y  Standard 4-3.5 of A.B.A Criminal Justice Standards (Defense Function)
     Model Rules 3.8 -especially Comment. Prosecutor has the responsibility of seeking justice, procedural and otherwise Same thing is found in ABA crim standards 3.12© And 3.13-F



 Session 6.2 (CRIMINAL CASES ± PROSECUTORS)
y Appearance matters j ³The appearance of justice is as important as justice.´  Appearance of impropriety isn¶t an acceptable factor, but the appearance of public figures is still important. j Public power j Government lawyers and judges Sources of rules: j Sources of rules especially applicable to government lawyers:  MRPC Rules 1.11, 1.12, 3.8  Official policies  Court decisions j Constitutions (state and federal)

y

 Young v. United States ex rel. Vuitton Et Fils S.A. y FACTS: Vuitton (P) filed a copyright infringement action against Young (D). P obtained a cease-and-desist order. Believing D to have violated his order, the district court charged him with criminal contempt. Counsel for P was appointed prosecutor. D was convicted and the court of appeals confirmed. y RULE: Counsel for a party that is a beneficiary of a court order may not be appointed as a prosecutor in a contempt action alleging a violation of order.  Allowing such would create opportunities for conflicts to arise and create the appearance of impropriety. y RATIONALE: The role of the criminal prosecutor is not to convict people but to seek justice. For that reason, prosecutors are forbidden by both federal law and professional ethics from representing the government in which they, their family, or their business associates have an interest. Therefore, a prosecutor in a criminal contempt matter cannot have an interest in the order upon which the contempt is based. Here, counsel for P the beneficiary of the allegedly violated order, was appointed prosecutor. P plainly had an interest in seeing the order enforced. Counsel was therefore placed in a position of serving two masters, justice and the P. this was improper and mandates a new trial. y Model Code: The responsibility of a public prosecutor differs from that of the usual advocate; his duty is to seek justice, not merely convict. j Prosecutors are ethically bound not to prosecute unless they are convinced of guilt. y Federal Prosecutors are prohibited form representing the Government in any matter in which they, their family or their business associates have any interest. y Private attorneys appointed to prosecute represent the US.

y

Comment 1 to 3. 8 y 'this responsibility to administer justice carries responsibilities to see defendant «.. These words although ideals go to heart of the ability of our social system to function, y Prosecutor has a fiduciary duty to state & people y Obligation to D's to make sure prosecution is fair  Might relationship b/w prosecutor and employee also create a problem in terms of conflict of interest? y Yes. Ct says,

y

Notice in Rule 1.7(a)(2), there's a concurrent conflict of interest on part of lawyer if htereis significant risk that one or more clients will be severly affected to the responsibilities of a 3rd person. y Who is 3rd person? Mother? Defendant?..... y U.S> has imposed on states has commanded the 'appearance of impropriety' 1.7 (a) (2)--> suggests that third person can be defendant? Ct. of appeals of ny refers to 'public confidence' -appareances of impropriety errode that confidenc.e Rule 1.7(a)(1). Responsibilty to protect corporaiton from fines, penalties, whatever y Important decisions that a prosecutor must make and appear to the public to make based solely on the values of the justice system: o Whether to charge o What to charge o Whether to offer immunity o Whether to bargain and what plea to accept o What sentence to seek y Vuitton¶s counsel were appointed to prosecute the contempt citation of individuals alleged to have violated a court injunction restraining certain trademark violations. y The Supreme Court¶s reversal is based on its supervisory power- they said a private lawyer should not prosecute on behalf of the public when the lawyer represents the private party in a related matter. o Blackmun: due process o Scalia: Separation of powers: judges can¶t appoint prosecutors y Court does not identify prosecutorial impropriety y Decision is prophylactic y Risks identified by the court are all about temptation to use prosecutorial power for private ends and the threat to the appearance of justice.
y

³Contributions to Justice´ (p. 231) y Extant Technologies believes it has been the victim of theft of secret technology, a crime. y The DA lacks the financial and technical resources to look into the case. y It asks the DA to investigate and offers to pay for experts or to provide its personnel to explain the technology and testify at trial. y The DA has four questions: o Any ethical reason why I cannot accept?  There are no rules that forbid this. o If I accept, is one choice better than the other?  Better to accept the in-kind help than the money. It¶s closest to what victims of crime regularly do. Victims of crime often provide info that will help in the investigation and prosecution. The testimony could be questioned, but that is a strategic not ethical issue. o If I accept, should I have any understanding with Extant?

If he accepts an offer of in-kind help, no special understanding is likely required. But if it is substantial and very valuable professional time, he may want to state that he is exercising independent judgment and can make no promises. o Even if I can do it, should I?  He should bc it is important to enforce law and he would not be able to do it without the help. y The problem is loosely based on People v. Eubanks (CA 1997): o ³[T]he courts, the public and the individual are entitled to rest assured that the public prosecutor¶s discretionary choices will be unaffected by private interests, and will be µborn of objective and impartial consideration of each individual case¶´ o The court disqualified the DA¶s office after it asked the company to pay an expert¶s substantial invoice and the company did. The court perceived a resulting sense of obligation that could skew prosecutorial decisions. In this case, the debt was already incurred.   People v. Superior Court y FACTS: Man was killed and his wife and another man were charged with this murder. The victim of the murder¶s mom worked in the District Attorneys office that was prosecuting the D¶s. The mom was to be a witness at the trial and stands to gain custody of the wife¶s child (mom¶s granddaughter) if she is found guilty. The child was given to the custody of the mom after the police consulted with the DA¶s office. The trial court concluded that these circumstances mandate that the DA should be disqualified. y RULE: A trial judge may disqualify a DA from participating in the prosecution of a D when the judge determines that the DA suffers from a conflict of interest that might prejudice him against the accused or may affect or appear to affect his ability to impartially perform the duties of his office. y RATIONALE: Court worried about the Appearance of Impropriety: y Social ± Need Appearance of Impartiality for Public Confidence in the Justice System. y Defendant ± A Fair and Impartial Trial is a Fundamental Right (5th & 14th Amendments).  People v. Zimmer  FACTS: Zimmer (D) was the organizer and manager of a business, Zimmer, Inc. For years he managed this corporation with no reins being placed on his managerial conduct by the investors. During this regime, no shareholders meetings were ever held and in the midst of a corporate financial crisis, he elected to surrender his corporate holdings and resign. The dissatisfied stockholders retained the District Attorney for the county as corporate counsel, who was also a shareholder in the corporation. The DA, while still being corporate counsel and a shareholder, brought charges against Zimmer and trial ensued. Zimmer argued that this was a conflict of interest and the DA should not be allowed to try him.  RULE: A District Attorney cannot bring suit against the former manager of a corporation for which he is also corporate counsel and a shareholder. He must recuse himself.  RATIONALE: The presumption of impartiality is undermined when there is a clear conflict of interest. The District Attorney¶s mission is not to convict but to achieve a just result. This is important because DA¶s have a wide latitude to operate: they decide whether to try the case, whether to offer a plea deal, what charges to bring, etc. y  Commonwealth v. Ellis y FACTS: By Massachusetts statute, the Automobile Insurers Bureau of Massachusetts was formed. The AIB is a voluntary, non-profit association of MA¶s automobile insurance companies which represents the interests of those companies in regulatory and rate-making



y

y

y

proceedings. Also, the IFB, a private investigative agency, was established for the prevention and investigation of fraudulent insurance transactions. At its outset the IFB was paid for by the AIB. A later statute mandated that the Office of the Attorney General should designate attorney¶s to work with the IFB in investigating and prosecuting fraud matters. The AIB not only provided money to the IFB but also indirectly to the Attorney General¶s office. Defendants who were indicted for insurance fraud moved to dismiss indictments or to disqualify Attorney General's office from prosecuting indictments on grounds that such office was not a disinterested prosecutor. The Superior Court denied motion. Granting application for direct appellate review, the Supreme Court held that: (1) statutes providing for insurance company underwriting of insurance fraud investigations and prosecutions, and for referral of such cases to the Attorney General's office by an Insurance Fraud Bureau (IFB) financed by insurers, does not compromise disinterestedness of prosecutors so as to violate due process principles; and (2) defendants failed to demonstrate that statutes violated due process as applied to them. RULE: In the absence of evidence showing that a prosecutor was not disinterested in a criminal proceeding, any inquiry into why a prosecutor exercised his or her discretion to prosecute would be inappropriate. The prosecutor must retain total control over the course of the investigation and all discretionary decisions. o A victim¶s direct funding of substantial expenses of a prosecutor¶s office would raise a question of control because the prosecutor may lose or appear to lose his impartiality because he may be beholden tote victim for assisting him. RATIONALE: The D¶s argued that the prosecutor did not have complete control because his office was funded by the insurance agencies. The Court found that this was not true and that it is in the public interest that victims and others expend their time, efforts, and resources to aid public prosecutors. The Court said that this financial influence was a ³paper tiger´ because if the insurance companies threatened or did actually withdraw their funding, the DA¶s office could get it from numerous other places. So in essence this would not be a viable threat. j Footnote 2: lawyers immediately stopped from practicing law upon charge of crime.

y

Supreme court of Mass. Says we're dealing with an appaerance of impropriety, which is a facial challenge y Two types of links between fraud division and insurance industry that are potentially problematic 1. Funding mechanism/ Collaboration a. No appearance of impropriety. 2. Powers Legislature has delegated to the private entity certain powers with regard to the funding mechanism between

   Standards 3-1.3, 3-3.8 and 3-3.9 of the A.B.A Criminal Justice Standards (Prosecution Function)
Pg 440

 Session 7.1 CLIENT CLIENT CONFLICTS ± CIVIL CASES
1. Civil Cases y ³Will You Represent Us?´ - Can Shiela¶s public interest office represent Bill and Miguel, who believe that the promotion policies of their employer, Beware Industries, discriminate and that as a result a white person with less seniority got a supervisor¶s job each one wanted?

y

-

What are Bill and Miguel¶s common interests? Do any of their interests differ? Is it possible to structure Sheila¶s work to avoid conflicts? What are the considerations?  Rule 1.2(c) A lawyer may limit scope of representation if the limitation is reasonable under the circumstances and client gives informed consent. ---Comment 7: ³although Rule affords lawyer and client substantial latitude to limit the representation, the limitation must be reasonable under the circumstances«although an agreement for a limited representation does not exempt a lawyer from the duty to provide competent representation, the limitation is a factor to be considered when determining legal knowledge, skill, thoroughness, and preparation reasonably necessary for representations (See Rule 1.1) Rule 1.7(a)(1/2) representing one client will be directly adverse to another client, or, a significant risk that the representation of one or more clients will be materially limited by the lawyer¶s responsibilities to another client, former client, or 3rd person, or by personal interest of lawyer. What consents are needed? Shiela could negotiate the settlements for both as long as non-settlement is premised on the assumption that one and not the other would get the job. She has to ensure the remedies she is seeking for them are acceptable to both of them.

  State ex rel. Morgan Stanley & Co., Inc. v. MacQueen y FACTS: The State has brought charges against D¶s alleging that there were some misdeeds in operating an investment fund. The State hired a private law firm, Wolf Ardis, to represent them. Morgan Stanley wanted to disqualify WA because they are representing the state and at the same time representing seven employees of the state, who are not named as D¶s but are in the complaint as engaging in some wrongdoing. The State wants to depose these seven. The Supreme Court held that: (1) conflict of interest existed, and (2) law firm could represent both State and employees if it amended its complaint to remove any allegations against those employees. y RULE: Attorney would not be disqualified for representing both state and state employees in state's action against business to recover state investment funds which were allegedly lost due to conspiracy between state employees and business, if state amended its complaint to remove any allegations against employees, who were not named as actual defendants in lawsuit. Rule 1.7(a). y RULE: The state cannot consent to dual representation. y RATIONALE: y In Conflicts Analysis ± the critical issue is the existence or potentiality of conflicts and not the inclusion of all adverse parties in a lawsuit. y The Court found that being named as a party to a lawsuit is not a prerequisite to creating the direct adversity element needed to establish a conflict under Rule 1.7(a). y Court asserted that the pleadings implicate the seven state employees and this is an adverse interest. Therefore, the conflict violated the law and the complaint must be amended to remove the allegations of these employees (instead of disqualification). - Rule 1.13e: sets up different rules from 1.7 when client is an organization.

A prerequisite to establishing a conflict of interest pursuant to Rule 1.7(a) of the rules of professional conduct is evidence that an attorney¶s representation of one client is directly adverse to another client. - The state fails to grasp that the consent required by Rules 1.7 and 1.13 is that of individuals and the state. state incapable of granting consent. Holding: pleadings drafted by Plaintiffs which refer to certain individuals in connection with specific allegations of wrongdoing but fail to include these individuals as defendants can contribute to a finding of adverse interests.  Consent required by Rule 1.7 is that of individuals and State, and state cannot grant consent.  There must be a showing of evidence that an attorney¶s representation of one client is directly adverse to another client. Wolf Ardis can only prove its case by implicating other clients, and the adversity does not require the staff members to be sued as individual defendants.

-

HYPO: Can you represent a couple that wants to adopt a baby and the biological mother form whom they adopt?  Rule 1.4 (b) ± Need to explain to client so that client can make informed consent.  Are these adverse interests? At present no but possibly in the future.  Say that even though it was not asserted directly against the other staff members, it creates a potential conflict between state and staff members, and then creates an apparent, if not actual, conflict of interest.  State opts to eliminate these seven individuals from the pleadings through amendment. If the conflict is no longer there by amending pleadings, how come Ardis can only still represent one group or the other? if there is no longer direct adversity (1.7a1), then why Ardis not represent both: it says it can only represent one if the pleadings are amended, and if they amend it, there is no direct adversity. Perhaps the former client¶s confidences. 1.79b4)«state cannot give consent because state has a duty to uphold public confidences. 1.7Comment 24 -a lawyer may take different positions. Conflict exists if there is a significant risk that a lawyers action on behalf of one client will materially limit the lawyer¶s effectiveness in representing another client *positional conflicts of interest; position with one client is different than that of another client. Comment for 1.7 and how we apply it, how is it to be applied? IMPUTATION OF CONFLICTS OF INTEREST RULE 1.10 ± Such situations can be considered from the premise that a firm of lawyers is essentially one lawyer for the purposes of the rules governing loyalty to the client, or from the premise that each lawyer is vicariously bound by the obligation of loyalty owed by each lawyer with whom the lawyer is associated. y If a lawyer is not allowed to represent a client due to conflict of interest rules then the whole firm is also disqualified from representation. y With some exceptions, mainly areas of government law offices, former government lawyers, and successive representations, both Code and Rules impute conflicts among all affiliated partners. Rule 1.10(a) y Lawyers are affiliated for imputation purposes if they work in the same office, regardless of title

y

Imputation has limits, Rule 1.10(a), which imputes Rule 1.7 and Rule 109, excludes imputation when one lawyer¶s conflict is based on personal interests if there is no µsignificant risk¶ that the representation will be µmaterially¶ limited.

y  Fiandaca v. Cunningham  FACTS: A class of P¶s comprised of female inmates of the NH (D) penitentiary system brought action in which they claimed that they were denied equal protection because male inmates enjoyed superior facilities. They were represented by New Hampshire Legal Assistance, a public-interest organization. NHLA also represented a class of students at a state school in an unrelated matter. At one point the D offered to convert one of the school buildings into a penitentiary for women. The students represented by NHLA vehemently opposed this, and the convict P¶s rejected the offer. A trial ensued, and the district court held that the D denied equal protection to female convicts and ordered that a facility be built. The D appealed, contending that the Court should have disqualified NHLA form representing the P class. RULE: An attorney may not represent two clients when a settlement offer made to one is contrary to the interests of the other.  RATIONALE: Model Rule 1.7 prohibits an attorney form representing a client if the representation of that client may be materially limited by the lawyer¶s responsibilities to another client. Thus, when a settlement offer is made and a lawyer owes allegiance to a party opposed to that settlement, that lawyer cannot use his independent judgment in advising his client. At this point a conflict exists. Here, NHLA could not recommend the D¶s settlement offer because of a duty to another client, and it therefore should have been disqualified.  It is usually possible for a potential conflict to be waived. This requires: (1) a reasonable belief by the attorney that he can zealously represent both interests, and (2) a knowing consent by the affected parties. The Court here however believed the conflict to be real and not potential. y Equal Protection claim challenging the facilities available to female inmates. y NHLA represents the class. y NHLA also represents the Garrity class, which was challenging conditions at Laconia State School. y On the eve of trial, the state offers to use Speare College at LSS for female prisoners. y NHLA rejects the offer and the state moves to disqualify it. j Argument is the lawyers represent both classes and can¶t make good judgment about what is the best deal for both. y NHLA¶s duties to the Fiandaca class y NHLA¶s duties to the Garrity class Court: ³It is inconceivable that NHLA, or any other counsel, could have properly performed the role of µadvocate¶ for both and the Garrity class, regardless of its good faith or high intentions. Indeed, this is precisely the sort of situation that Rule 1.7 is designed to prevent.´ : Dist. Ct. Erred in permitting NHLA as ¶s counsel after trial had become apparent --Disqualification pursuant to Rule 1.7 CT: NHLA owes plaintiffs undivided loyalty, and to ensure that the offer received full and fair consideration by members of the class j y But what remedy? j The case had already been tried in lower court, so the higher court said they wouldn¶t reverse. j NHLA didn¶t do anything in the trial that appeared to be a conflict. j The lower court judge refused to award the remedy of Speare Cottage.

 1.4a/b, 1.2a, 1.8g, all of which say the client makes the ultimate decision.  Re; mentally retarded, rule 1.14.  Pg 243: there is at least the appearance of having been tainted by NHLA¶s conflict of interest. This smacks of an appearance of impropriety.   1.6b: a lawyer may reveal (not must)«to the extent that the lawyer reasonably believes necessary.  4.1(b):   Hypo: if you work for government agency, and you find some statute is being violated or objectionably effected, can you sue the government? Could the DA sue the county in a civil matter?  What if a gov¶t attorney is a volunteer for a volunteer legal group, can government represent environmental group in blocking the environmental license. j Imputation (Rule 1.10(a): ³While lawyers are associated in a firm, none of them shall knowingly represent a client when any one of them practicing alone would be prohibited from doing so by rules 1.7 or 1.9 unless the prohibition is based on a personal interest of the prohibited lawyer and does not present a significant risk of materially limiting the representation of the client by the remaining lawyers in the firm.´ o Courts have carved out an exception for legal services and public defender offices. These lawyers are salaried.

 Session 7.2
 North Carolina State Bar v. Nifong  Benjamin Weiser, Doubting Case, A Prosecutor Helped the Defense

 Session 8.1
 Simpson v. James  FACTS: Simpson (P) operated a restaurant, which she desired to sell. She contacted James (D), an attorney who had previously represented her, concerning a sale. Oliver (D), James Partner, facilitated a transaction between P and Tide Creek, Inc. A deal was arranged wherein the business was sold for $500,000, $100,000 of which was paid down, with $400,000 payable in notes, secured by TC, Inc. stock. The business proved unprofitable for TC, which eventually went bankrupt. P sued D¶s, who had represented both sides in the transaction, contending that it had constituted malpractice for them to do so. A jury awarded P $200,000 and an appeal was taken.  RULE: An attorney may commit malpractice by representing both sides in a transaction.  RATIONALE: While representing both sides is not inherently impermissible, it may make it difficult for an attorney to represent one side, the other, or both as zealously as professional standards require.  y Tide creek, the purchaser, is devoid of assets, and Simpson is trying to recoup as much as possible. Sues law firm for malpractice. If he did not fulfill the fiduciary duty, he will be        Did a lawyer-client relationship come into existence between James and Simpson @ January 9th meeting? Yes. James argued the opposite. Simpson apparently asked him to represent her, never gave her advice or time. The test seems to be what the client reasonably believed. What happened after tide-creek purchased HP enterprises? The firm continued to do estate and tax work for Simpson. So firm was representing her. Model Rule 1.10 cmt 2. Second sentence. Did Mr. James mislead Simpson to the point that he violated professional ethics? What did he say to her? Rule 4.1 Did james violate rule 4.1?

Jan 29th he encouraged her about the viability. She added that she relied on those assurances. What is 4.1 getting at: False statements of material fact. Were those statements factual? Probably not, because they were opinion, but it can be argued either way.  Were those statements material?  4.1-2 applies to facts, not to opinions. Is that fact or opinion?  What is the pertinence of the conflict of interest? is it at all relevant to liability? Arguably no. because if everything had been done well, there would have been no liability or neg. But the conflict of interest shows that there is something that should have been done or not done. Conflict of interest made it appear and easier to demonstrate that something wasn¶t right. y Simpson and the Joneses sold their catfish restaurant to Tide Creek. y Buyers and sellers were represented by Ed Oliver, who had long represented the restaurants when run by Simpson¶s late husband. y Oliver got the buyer¶s personal guarantees on a note, a lien on the stock of Tide Creek, and restrictions on the operation of the business (good things that a seller would want). o He didn¶t get a lien on the inventory or any interest on insurance proceeds. y Tide Creek had an insurance loss and David James of Oliver¶s (now former) firm helped it collect $200K in insurance. y Tide Creek defaulted on a $200K note, Simpson consulted James about her options. y Tide Creek then declared bankruptcy. Simpson¶s note is worthless. y Simpson sues James and his firm. What is her theory of liability? o Malpractice, negligence y What did Oliver and James do wrong and what does the alleged conflict have to do with it? o Either the lawyers were guilty of malpractice (negligence) or they weren¶t. o Theory: the conflict enabled ¶s lawyers to explain to the jury why the failed to get better protection. In effect, it shifted the burden of proof to to show that an unconflicted lawyer would have fared no better. o ¶ lawyers love conflicts. Why? y Is it bc of burden shifting? y James¶s defense: I did not represent Simpson on the note. Not plausible. y Could the law firm have gotten informed consent for the multiple representations? o Generally not a good idea unless the client interests are strongly aligned, not so here. o Informed consent requires information and the amount of information depends on the client¶s sophistication. o The value of independent counsel. y Is it possible to have informed consent to a conflict that has not yet arisen, whose details cannot be foreseen, and which may never arise? YES. Rule 1.7 comment 22 Restatement § 122 comment d Considerations: sophistication of client, independent counsel, informed to the risks.      Public Service Mutual Insurance v. Goldfarb  FACTS: Goldfarb (D), a dentist, was accused by one of his patients of sexually assaulting her while she was under sedation. Criminal charges were filed, and he was convicted. The victim filed a civil suit against D, which he tendered to his professional liability carrier, Public Service Mutual (P). P responded by seeking a declaration that the alleged act did not trigger a duty to defend and indemnify D.  RULE: Sexual assault by a doctor against a patient may trigger a professional liability carrier¶s duty to defend and indemnify in a subsequent civil suit.



   

  

RATIONALE: To the extent that a jury in a civil suit finds that injury upon the victim was unintentionally caused, the duty to indemnify will arise. Since this determination cannot be mad prior to trial, P must defend D for as long as the potential of a duty to indemnify exists, the duty to defend also exists. The reasoning used in arriving at the rule here is fairly universal. The duty to defend arises when any aspect of an action might be covered. The terminology used is ³the duty to defend is broader than the duty to indemnify.´ PSMI: what creates a conflict of interst here? Recovery of one would be the detriment to the other. Where is the conflict of interest? Whether the conduct intended to harm. Damages are pursuant to this. If the injury was negligently sustained, then that would mean the policy provided coverage. If it was intended, then it wouldn¶t cover. Thus, the lawyer is trying to argue that they are intended acts. Goldfarb wants to say these were not intentionally inflicted injuries. One lawyer, a lawyer representing the insurance company and Goldfarb, is an impossible situation. Court of App. Says when you have a situation like this, where the interest of the insured diverge from the insurance company. Dentists must have a lawyer paid for by insurance company that represents the dentists interests. The alleged injuries may be covered by conduct of the policies. Hypo: plaintiff sues defendant for injuries for car wreck. D has two insurance policies. One for car and one for home. Company A and B represent each. Company A says we¶re not going to get involved in this b/c it is a matter of the car, not a house. Plaintiff was saying it was injured b/c the problems with the car and the driveway(home). So homeowners insurance company is trying to get out of case. Can one attorney represent both the D, company A and company B? Defendant wants to have both representations of a and b. but he¶ll need the third. Company relies on Goldfarb to get to this conclusion. HOA Ins. Said if we lose this suit, we want our counsel present. Having abandoned thehomeowner, you¶re simply going to go unrepresented. Having breached its policy, new York court did not look kindly on the hoa ins. Co.

    Allstate Ins. Co. v. Aetna Cas. & Sur. Co.  FACTS: Linda Friedman was driving her neighbors car and injured Yvonne Goellar. The neighbors were insured by P and P assumed complete control of Friedman¶s defense. P learned two years later that Friedman was insured by D. P lost the case against Goellar and after the case sued D to recoup their losses. The Court found for D, Aetna.  RULE: Where a conflict of interest between the insured and the insurer requires the insured to hire independent counsel the insurer has a contractual duty to pay to defend the lawsuit.  RATIONALE: Court found that by the terms of the insurance policy that P was obligated to pay to defend Friedman. The Court was swayed by the two year interval from the accident to when D was told of the accident. Court found that P¶s two year delay in informing D of the accident and resulting action prejudiced D¶s rights. y  Spaulding v. Zimmerman  FACTS: P was injured in an automobile accident where the D was the driver. P had lots of injuries and went to the doctor. Before settlement, D wanted P to see a doctor of D¶s choosing. P went to this doctor. This doctor concurred with the other doctors and in a report to the D¶s counsel informed them that P also had aortic aneurysm, which is a life threatening condition mandating surgery to repair. P did not know of this condition and the D or his counsel never informed him. The sides settled for a far less amount of money than had the P known of the aneurysm. Later, the P undertook a routine physical where the aneurysm was discovered. The Doctor went back to the original X-Rays and verified that



   

the aneurysm was from the accident. P sued D to nullify the original settlement. The Court found for the P. RULE: The court may vacate settlement which court has approved on behalf of minor suing for injuries for mistake even though mistake was not mutual in sense that both parties were similarly mistaken as to nature and extent of minor's injuries, but where it is shown that one of parties had additional knowledge with respect thereto and was aware that neither the court nor the adversary party possessed such knowledge when the settlement was approved. MODEL RULES: RULE 1.6 CONFIDENTIALITY OF INFORMATION: A lawyer may reveal information relating to the representation of a client to the extent the lawyer reasonably believes necessary (1) to prevent reasonably certain death or substantial bodily harm RULE 3.3 CANDOR TOWARD THE TRIBUNAL RULE 4.1 TRUTHFULNESS IN STATEMENTS TO OTHERS ± A lawyer shall not knowingly make a false statement of material fact to a 3rd person. y Spaulding y -doesn¶t find out about aneurysm until years later that his doctors believed resulted from accident. He goes back and wants additional damages b/c the attorneys weren¶t open b/c they didn¶t reveal this material fact for a far more favorable settlement. This is a case before MR. y y Updating the case, what rules would be applicable to the attorney for the defendants that say they were permitted to disclose evidence? y y 1.6b1/1.6a«a lawyer may reveal to representation to prevent reasonably certain death or bodily harm. This presupposed that it is reasonably certain y y What is meant, µto prevent reasonably certain death or substantial bodily harm¶? y Paragraph 1-6-16 of Scope: rules do not exhaust y Must the lawyer for the defendants disclose under the MR? y Rule 3.3, candor to the tribunal. What part: Section a1. Shall not knowing make a false statement of fact or tribunal. If you look at comments, even failure to disclose would be misrepresentation to court. They knew about it and signed off. That would be considered. Comment 3. Attorney simply sat silent and said nothing. No affirmative statement. y y Rule 3.3c²applies throughout court proceedings. Apply under Paragraph A. Notice under 3.3c. Even if compliance requires violation of rule 3.6 Confidentiality is superseded by candor to the tribunal. y y Rule 8.4c/d. Conflict dishonesty deceit or misrepresentation. y y Rule 1.2d. a lawyer should not counsel/assist a client in conduct that a lawyer knows is fraudulent. 8.4 does cover deceit. So this is a type of fraud, and thus within 1.2d as well. A statement containing a half-truth is the same as a false statement. «that undisclosed facts«.and it is immaterial as to whether it is relevant to the value of it. y y Also know rule 1.2 comment 10. Pg 14. ³may not continue assisting´«lawyer must withdraw from representation. IN some cases, withdrawal alone may be insufficient. Must make a noisy withdrawal y

y

1.2d«.conduct that the lawyer knows is criminal«.is there a crime here? No disclosure? Might there be a crime? Reckless endangerment. A person is guilty of reckless endg if with indifference to human life, acts with reckless

Rule 4.1. in course of representing client, a lawyer shall not make false statement of material fact. Now if the lawyer for the defendants has made court to believe. Might that third person. Comment 1. y 4.1b. in course of rep. not knowingly y 4.1b1. does 4.1b require disclosure ? Udner 4.1, under cmt 3. Which says, in extreme cases, a lawyer may be required to disclose j  Fickett v. Superior Court (Client-Third Party Conflicts) y Contract of employment was between guardian of estate of ward and attorney. But privity of contrat was present, but not to ward. What does the court conclude here/ they disagree with atty¶s argument. Court says there is a rebuttable presumption. Notice comment for rule 1.14. Comment 4. y Rule 1.2, comment 11. When lawyer is fiduciary, he has special obligations. y y Lawyer knew or should have known. So if the lawyer knew or should have known, the lawyer had an obligation b/c injury was foreseeable. y Factors 5 and 6 rare not explained. But court says with regard to 5 and 6, refers to atty having Imputed Fiduciary-Client Conflicts. A Minnesota disciplinary case and an Arizona malpractice case indicate that attorneys may bear responsibility for actions taken by fiduciaryclients which involve conflict or harm to the ward, conservatee, or other beneficiary. In Fickett v. Superior Court, 558 P.2d 988 (Ariz. 1976), a guardian misappropriated substantial funds from a ward. Fickett was the guardian¶s lawyer, but did not know of the misconduct. A successor guardian sued Fickett for negligence. The court affirmed denial of Fickett¶s summary judgment motion, holding: y [W]hen an attorney undertakes to represent the guardian of an incompetent, he assumes a relationship not only with the guardian but also with the ward. If [the lawyer] knew or should have known that the guardian was acting adversely to his ward¶s interests, the possibility of frustrating the whole purpose of the guardianship became foreseeable as did the possibility of injury to the ward. In fact, we conceive that the ward¶s interests overshadow those of the guardian. y The guardian¶s duty to the ward was in some sense imputed to Fickett, presumably because Fickett was representing the guardian not in the guardian¶s personal capacity, but in his fiduciary capacity  Fickett involved an action filed by the present conservator of an incompetent's estate against the former guardian and that guardian's attorneys, claiming that Fickett, as the attorney for the former guardian, had been negligent in failing to discover that the guardian had embarked upon a scheme to liquidate the guardianship estate by misappropriation and conversion of funds to his own use, and by making improper investments. The Pima County Superior Court denied summary judgment in favor of Fickett, rejecting the argument that absent a showing of fraud or collusion, the attorney owed no duty to the ward and a cause of action for malpractice could not be maintained. The Court of Appeals affirmed that determination, holding:  The general rule for many years has been that an attorney could not be liable to one other than his client in an action arising out of his professional duties, in the absence of fraud or collusion . . . In denying liability of the attorney to one not in privity of contract for the consequences of professional negligence, the courts have relied principally on two arguments: (1) that to allow such liability would deprive the parties to the y

y y

 









contract of control of their own agreement; and (2) that a duty to the general public would impose a huge potential burden of liability on the contracting parties. ****** We cannot agree with petitioners that they owed no duty to the ward and that her conservator could not maintain an action because of lack of privity of contract. We are of the opinion that the better view is that the determination of whether, in a specific case, the attorney will be held liable to a third person not in privity is a matter of policy and involves the balancing of various factors, among which are the extent to which the transaction was intended to affect the plaintiff, the foreseeability of harm to him, the degree of certainty that the plaintiff suffered injury, the closeness of the connection between the defendant's conduct and the injuries suffered, the moral blame attached to the defendant's conduct, and the policy of preventing future harm. Id., 27 Ariz. App. at 794-95, 558 P.2d at 989-90. The Court, rather than abolishing a privity requirement entirely, however, allowed the action by the successor conservator to proceed on the basis that when an attorney undertakes to represent the guardian of an incompetent, that attorney assumes a relationship not only with the guardian but also with the ward. That rationale is consistent with a later holding that an attorney for the Special Administrator of an estate has a "derivative fiduciary duty" to the successors of the estate. Matter of Estate of Shano, 177 Ariz. 550, 869 P.2d 1203 (App. 1993). The so-called "Fickett test" was subsequently applied, however, to deny standing to a workmen's compensation carrier to pursue a malpractice claim against the attorney for parties to whom it was paying benefits for his failure to file an action against a potentially liable third party before the expiration of the statute of limitations. Travelers Insurance Company v. Breese, 138 Ariz. 508, 675 P.2d 1327 (App. 1983). The Court's rationale was that the carrier, Travelers, was not intended to be the prime beneficiary of the attorney-client relationship between the attorney and the compensation claimants, and it was not foreseeable that the attorney's failure to file the third-party action on time would prejudice any rights of Travelers. Subsequently, in Donnelly Construction Co. v. Oberg/Hunt/Gilleland, 139 Ariz. 184, 677 P.2d 1292 (1984), the Supreme Court expressly rejected the imposition of any requirement of privity to maintain an action for professional negligence, and had disapproved prior decisions, including Chalpin v. Brennan, supra, which had purported to impose one. The only appellate decision dealing with the impact of Donnelly on legal malpractice claims is Franko v. Mitchell, 158 Ariz. 391, 762 P.2d 1345 (App. 1988), which was an appeal from a summary judgment entered in favor of a lawyer, Mitchell, on breach of contract and legal malpractice claims brought by a Ms. Franko. Franko had agreed to loan her boyfriend, Markoff, a sum of money, and Markoff had asked Mitchell to draw up the promissory note evidencing the obligation to Franko. Both Markoff and Franko went to Mitchell's office to review and sign the note. Markoff subsequently defaulted and disappeared. The Court held that there was a genuine issue of material fact as to whether an attorney-client relationship had been formed between Mitchell and Franko, which required the reversal of the summary judgment entered below. The Court then went on to discuss Franko's contention that she was entitled to bring a legal malpractice action against Mitchell even if she was not his client. Noting that the Supreme Court had in Donnelly abolished any requirement of privity in professional negligence cases, the Court ruled that Franko's ability to pursue malpractice claims against Mitchell would turn on the application of the test announced in Fickett v. Superior Court, supra. The Court then offered the following explanatory comment concerning the "Fickett test": We are of the opinion, however, that the test utilized in Fickett does not create a distinct duty of care towards a third party, as Franko suggests, but instead allows a third party in certain situations to sue an attorney for negligence to his client. That is, under the Fickett test any duty owed by an attorney to a third party is derivative of the duty owed by that attorney to his client. y

------------------------------------------------------------------

THE ADVOCATE WITNESS RULE A special conflict confronts attorneys who are or ought to be called as witnesses in a litigation in which they represent one of the parties. y Model Rule 3.7 ± does not distinguish between testimony for or against a client. Unless one of the narrow exceptions applies, it simply prohibits lawyers from acting ³as advocate at trial if the lawyer is likely to be a necessary witness.´ o This disqualification runs only to advocacy at trial not pre-trial work. o Other lawyers at the witness/lawyers firm are not imputed from representing the client so long as they do not have conflict of interest issues. y Policies Behind the Advocate-Witness Rule: o The Jury may accord the lawyer¶s testimony to much weight because their special knowledge of the case. o Professional courtesy may restrict the opposing counsel on cross examination. o Possibility that the testimony would degrade the public¶s view of the legal system. Lawyer might not be able to distinguish between his role as advocate and witness ----------------------------------------------------------------------------

 Session 8.2: Successive Conflicts of Interest
 Analytica, Inc. v. NPD Research, Inc. y FACTS: Malec was an employee of NPD Research (D) who was given an equity interest as compensation for certain services. The law firm of Schwartz & Freeman (SF) handled the transaction. Subsequent to this, Malec left D, forming Analytical, Inc. (P), which established itself as a competitor of D in market research. P subsequently filed an anti-trust action against D. P was represented by SF. D moved to disqualify SF. The district court granted the motion, and P appealed. y y RULE: A lawyer may not represent an adversary of his former client if the subject matter of the two representations is substantially related. This is the Substantial Relations Test RULE 1.9 (a) (2 Part Test): o (1) Is matter Substantially Related?  Lawyer could have attained confidential information in the 1st representation that could be relevant in the 2nd.  The Model Rules don¶t mandate that the information even has to be confidential; just that it is relevant in both suits.  If YES Then it is Substantially Related. o (2) Are Positions Materially Adverse?  Look at Rule 1.7 (a)(2) o **If there is a Substantial Relationship Need consent from both former and current client to continue in capacity as lawyer. ***Rules 1.7 and 1.9 make it important to determine if client is a former client or a current client. ****Remember too that the lawyer¶s conflict will be imputed to all of her office colleagues. Rule 1.10 (a). RATIONALE: Specifically, a lawyer may not represent an adversary of his former client if the subject matter of the two representations is substantially related. If confidential information that might have been obtained during a representation might be relevant in the second, then the attorney must be disqualified. The fact that the attorney might not have actually attained the information is of no consequence. A per se rule of disqualification is

y y

y

preferable, as determination of the facts underlying a motion to disqualify would be difficult and time consuming if a case-by-case analysis were employed. Here, SF represented D in a financial transaction, and then represented an adversary in an anti-trust suit. It seems clear that information that the firm might have obtained during its representation of D might be relevant in the present action so disqualification is proper. AFFIRMED. y The rule stated here is universal. In the current age of megafirms, it sometimes presents a problem in that disqualification of one member of a firm usually disqualifies the entire firm. A large firm might have many clients and the possibilities of inadvertent conflicts are ever present.

y

MALPRACTICE BASED ON SUCCESSIVE CONFLICTS ± A law firm that acts adversely to a former client in violation of the substantial relationship test will subject itself to liability for breach of fiduciary duty.

RULE 1.16 DECLINING OR TERMINATING REPRESENTATION ± deals with how an attorney can get out of representing a client.  What¶s the difference between them? analytica is more simpler, a one-part test. Lasalle has three parts. Analytica just looks at whether there is a possibility of the transfer of information, the lasalle test seems to emphasize looking at the facts of the first representation for judging whether there any confidential information that could have been acquired.  Comment 3, to 1.9: matters that are substantially related if they involve the same transaction or legal dispute or if there otherwise is a substantial risk that confidential factual information as would normally have been obtained int eh prior representation would materially advance«.   The question 1.9 poses in relation to 1.7, is when is a former client, a current client convert to a former client.   If former client, then 1.9 prevails, if current client, 1.7 prevails.  1.7a2.   1.3 Comment 4.  Analytica test:  LaSalle Test  A lawyer may not represent an adversary of his former client if the subject matter of the two representations is µsubstantially related.¶  Meaning: if the lawyer could have obtained confidential information in the first representation that would have been relevant in the second.  It is irrelevant whether or not information was obtained and/or used against the adversary.  Irrelevant whether the firm is a solo practitioner or a firm.  Exception: when a member or associate of a law firm (or gov¶t legal dept) changes jobs, and later he or his new firm is retained by an adversary of a client of his former firm. y Why: in such a case, even if there is a substantial relationship between the two matters, the lawyer can avoid disqualification by showing that effective measures were taken to prevent confidences from being received by whichever lawyers in the new firm are handling the new matter. y This exception is inapplicable in this case: Fine (the representative for the Malec¶s when the were with NPD and who later left to create Plaintiff/Analytica), had access to and received confidential financial operating data on the deal to transfer stocks. y

j The data concerned NPD¶s profitability, sales aspects, and general market strength all of which are pertinent to liability and damage phases of an antitrust suit charging against monopolization. Substantial Relationship Test: a factual inquiry in every case into whether confidences had actually been revealed would not be a satisfactory alternative, particularly in a case such as this where the issue is not just whether they have been revealed but also whether they will be revealed during a pending litigation. y MR 1.9(a): The modern articulation of the substantial relationship test. j ³where any substantial relationship can be shown between the subject matter of a former representation, the latter will be prohibited j ³«the former client need show no more than that the matters embraced within the pending suit wherein his former attorney appears on behalf of his adversary are substantially related to the matters or cause of action wherein the attorney previously represented him, the former client. j The court will assume that during the course of the former representation confidences were disclosed to the attorney bearing on the subject matter of the representation. j Posner on ³substantial relationship´: ³if a lawyer could have obtained confidential information in the first representation that would have been relevant in the second.´  When are two matters substantially related? (remember rule 1.10a imputes lawyers conflict to all other firm employees. If the former client can show that the new µadverse¶ matter is substantially related to the former matter (using language like the language in analytica or some variation), it is irrebutably presumed that the lawyer gained confidential information in the prior matter relevant to the new one. a. The Continuing Duty of Loyalty 1. Confidentiality is not the only goal of successive conflict rules 2. Another value the rule protects is loyalty to a former client as a way to encourage clients to repose trust in their lawyers during the professional relationship. rule forbids lawyers to switch sides and oppose a client even if not confidential information is at risk. 3. Notion that loyalty to a former client survives the termination of the relationship is carried over in the Model Rule 1.9 and its comment. The text of Rule 9.1(a), the embodiment of the substantial relationship test, does not depend on the existence of confidential information. 4. Hypo: when two business partners hire an attorney to jointly represent them in getting a deal done, and the deal falls out, and one client retains the attorney to represent him, and the other client objects, the lawyer/client can argue that the other client should have no expectation to the attorney-client privilege for joint representations. Christensen & Allagaert, refused to disqualify attorneys in positions like the lawyer, holding that the other client cannot complain: but this is a very narrow reading and under unusual facts where the other client should have anticipated that the lawyer would side with the other client in event of a dispute. Accomodation (Restatement), one who, under the circumstances, would have µunderstood and impliedly consented to the lawyer¶s continuing to represent the regular client in the matter.´ 5. When two Courts take the view that a duty of loyalty prevents the successive adverse representation

b. The Consequences of Disqualification y Model Rule 1.109a) disqualify all lawyers in a law office from opposing a client when any lawyer in the office has represented that client on a substantially related matter. y When a lawyer is disqualified, the client will have to get new counsel, who will wan tto receive the disqualified firms files. 6. Opposing party may still object to this as it gives the new firm the benefit of suspect work. Nevertheless, absent any identifiably tainted item, the courts have been disposed to allow turnover to successor counsel. 7. Texas SC has created a rebuttable presumption that the work product contains the confidential information«current client can overcome this presumption by demonstrating that there is not a substantial likelihood that the desired items of work product contain or reflect confidential information. c. Malpractice Based on Successive Conflicts y A law firm that acts adversely to a former client in violation of the substantial relationship test will subject itself to liability for breach of fiduciary duty, and at the very least, will violated the continuing duty of loyalty. y Of course, in a civil action, the client will have to prove damages. d. Who Is a Former Client? (read over!!!!!!) y A preliminary interview can create an attorney-client relationship. e. Like a Hot Potato y A client who wishes to disqualify a lawyer would prefer to be a current client, while the lawyer would prefer to say that the client is a former client (if ever a client at all). y Unified Sewerage Agency v. Jelco 8. µlaw firms could not escape the stricter current-client conflict rules simply by withdrawing from the representation and converting a current client into a former one. o protects a client¶s interests in uninterrupted representation to the conclusion of a matter. While a law firm may withdraw from a current representation, it may do so only for the reasons listed in Rule 1. 16. o The law firms own economic interests generally have not (despite ambiguity in rule 1.16(b1) and (b6)) been deemed an acceptable reason for dropping the client. o ³ a firm may not drop a client like a hot potato, especially if it is in order to keep happy a far more lucrative client´ 9. Is there anything a law firm can do to protect itself and other clients that may depend on it? o The advance consent from an incoming client that it will agree to terminate the relationship, and will not invoke the hot potato rule, in the event of adversity with a long standing firm client? ABA opinion 93-372 concludes that appropriately detailed advance consents can be proper. f. Standing and Waiver y Concurrent conflicts ma sometimes be waived. Successive conflicts may always be waived. (Rule 1.9(a). ) g. The Appearance of Impropriety y Pretty soon the appearance of impropriety standard became overused and the object of criticism, not least of all because of its unpredictability. 10. Restatement does not employ it 11. The Model Rules omit the appearance standard altogether. 12. But don¶t dismiss the µappearance test¶ altogether.

o Tens of thousands of now aging lawyers and judges have grown up with the appearance of impropriety etched in their consciousnesses. (Smell test) o Some courts may for a time continue to rely on the appearance of impropriety in disqualifying a lawyer o A couple of states retain the code of professional responsibility and its Cannon 9: while that doesn¶t mean that courts in those states will make judgments about conflicts based on the µappearance¶ language alone, the language may be part of court¶s reasoning. o Even states that have adopted the Model Rules may continue to look to appearances.  ³Although the new Rules do not use the language, the µappearance of impropriety¶ standard has not bee wholly abandoned in spirit.  New jersey made a point of retaining the µappearance¶ language in its version of the Model Rules. So it is worth remembering that appearances may still count, depending on where you are.  Finally, the µappearance of impropriety¶ standard continues to have a role in evaluating the conduct of public officials ± lawyers and judges¶ y  Schwartz v. Cortelloni  FACTS: Both parties maternal great aunt executed a will which granted a life estate in a plot of land to Malerich, with a remainder interest to Melerich¶s ³blood lineal descendants or descendant.´ At the time of the will Malerich had one child, Cortelloni (D). Malerich took possession of the land after the great aunt¶s death. In 1956, the Gehlbach law firm appeared on behalf of Malerich as guardian of D in a sale of real estate case. This real estate sale had no relation to the plot of land at issue in this case from the will. Later, Malerich gave birth to Schwarz (P) out of wedlock and gave her up for adoption. After Malerich¶s death, P learned of the will and retained Gehlbach as counsel to retain her ownership interests in the land. D moved to disqualify Gehlbach as counsel based on his prior representation of D and the substantial relationship between the two representations. The Court found that D failed to meet her burden of showing a substantial relationship between the matters involved in the two representations under Rule 1.9. Therefore, Gehlbach may continue on as P¶s attorney in this matter. y RULE: Court found that an attorney should not be disqualified from representing a client whose interests are adverse to a former client solely on the basis that the subsequent representation may create the appearance of impropriety. y RATIONALE: j Court Used the 3 Part LaSalle Test:  (1) Court must make a factual reconstruction of the scope of the former representation.  (2) It must determine whether it is reasonable to infer that the confidential information allegedly given would have been given to a lawyer representing a client in those matters.  (3) The Court must consider whether the information is relevant to the issues raised in the litigation pending against the former client. j Why did the Court not find a substantial relationship?  Time Period D was Gehlbach¶s client 40 years previously and had since not represented D.  No information that was not public knowledge was received by Gehlbach from first representing the D.

 Legal Ethics Opinion 1762 of the Virginia State Bar y FACTS: Mother (M) and her infant child (C) were involved in an automobile accident with an uninsured vehicle that was owned by the Mother. The M was operating her car with Passenger (P) in the front seat holding the child in her lap. The accident occurred injury M, C and P. M retained lawyer to represent her, suit was filed and lawyer obtained a substantial settlement for M for her injuries before the trial date. Now that the M has settled her case against the Tortfeasor, M has requested lawyer to present M¶s claim on behalf of and for the benefit of C to Tortfeasor¶s insurer. She wants lawyer to file suit for her on the C¶s behalf if necessary. y ISSUE: Can the lawyer ethically accept employment by the M to recover funds for C¶s personal injury claim against Tortfeasor? y RULE 5.4 3rd Party Payor (here the M) (c) ± The lawyer needs to pursue the objectives and interests of the child, not those of the M. The lawyer should not allow the M to ³direct or regulate the lawyer¶s professional judgment in rendering legal services.´ y Rule 1.9 Conflicts of Interest Between Current and Former Clients ± The lawyer In representation of a current client, may not be adverse to a former client in a substantially related matter, unless the former client and the new client consent to that new representation. o Since it¶s from the same accident the two matters are substantially related. o Is the representation adverse to lawyer¶s representation of the M?  If the lawyer has to file a claim against the mother the representation is adverse.  If the lawyer is only filing exclusively against the other driver the representation is not adverse.  Even if the representation is adverse, the lawyer could represent the C if the M and C consent (however, this brings up the question of if a child can consent and this is not part of this opinion, so it will not be answered).

y y

NEW RULE: Rule 1.7(a)(2): (put this in your outline!) Significant risk that the representation of one or more clients will be materially limited by the lawyer's responsibilities to another client, a former client, or a third person or by a personal interest of the lawyer.

y y y y y y y y y

Mother goes to lawyer to represent child in an action. Committee says we have to look at three roles of the mother (as she is paying the lawyer) (she¶s the next friend of the child) (former client of the lawyer) 5.4C And 1.8f Client is the child, not relevant that the lawyer is being paid by the mother. The client is the child. 5.4c and 1.8f recognize. And with regard as guardian/friend of child« How can lawyer get the consent of the child? Can¶t really. Do notice rule 1.7a2.


y  Greig v. Macy¶s Northeast, Inc. y FACTS: The P (Greig) retained the firm of Kenneth Weiner to represent her in a matter against D (Macy¶s) for allegedly being wrongfully targeted (racial discrimination) as a shoplifter while in a Macy¶s store. During this representation, the P and Weiner disagree and the Court grants Weiner¶s withdrawal from representing P. Then, P files an action for malpractice against Weiner. Weiner¶s malpractice carrier retained the Amdur firm to represent Weiner in this action against the P. Another lawyer from the Amdur firm already represented D (Macy¶s) in the initial

  

lawsuit. The P alleges that this dual representation constitutes a conflict of interest and creates an appearance of impropriety and that the firm should have to withdrawal as counsel for Weiner. j Greig¶s attorney is mr. weimer j Greig and weimer had a disagreement and j So former client becomes plaintiff against macy¶s and his now ex-attorney j Who was representing macys: amber firm. District court did not like the situation here. Notice how the court says at different points in the opinon, the rules technically don¶t apply here. We¶re invoking the spirit of the rules. 1.6b5. but only to the extent reasonably necessary to protect himself. How is the amdur firm whose violations of the rules are the basis for the disqualification. What rule is the amdur firm violating here? 8.4a. violating rules through someone else. y pg 152. 2nd prgrph. Rule 1.9a. why is that relevant? 1.9 does not literally apply here. Notice the sentence, ³the rules literally do not apply´ theyr¶e in the business of making justice. It was just unacceptable. They have to look upon these rules and statutes and provisions and realize that they are applied with considerable flexibility. 2nd Basis for disqualification of Amdur firm: appearance of impropriety. j The appearance of impropriety«why does it appear here? j If amdur acquires information from weimer, then it would give macy¶s an unfair advantage. That it would appear to be improper. Or, if amdur acquires the information but cannot use it then it would also appear to be improper. Greig case cites a case several times. Given the injury that mrs. Dewey would suffer, we would not disqualify. However, from this point forward, (1,800 hours in the case), the wilentz firm is working for nothing. For free. Trial court, if dewey prevails, will have to determine how much they should be compensated. Anything beyond that, go to mrs. Dewey.



 

RULE: APPLICABLE MODEL RULES: y 1.6 Duty of Confidentiality o (b)(5) ± a lawyer may reveal information relating to the representation of a client to establish a defense on behalf of the lawyer. o NJ-SC ± ³a lawyer sued for malpractice is obligated to reveal privileged information only to the extent necessary to establish a claim or defense on behalf of the lawyer in a controversy between the lawyer and the client. y 1.10 Imputation of Conflicts of Interest ± if one lawyer is disqualified, all lawyers at the firm are disqualified. y Rule 8.4 Misconduct ± It is professional misconduct for a lawyer to: o (a) violate or attempt to violate the Rules of Pro Conduct, knowingly assist or induce another to do so, or do so through the acts of another. Or to o (b) engage in conduct that is prejudicial to the administration of justice.  Ct. found that the Amdur firm violated this rule by representing Weiner because it should have been clear that Weiner would be in danger of violating his Duty of Confidentiality to Greig by divulging such information to Greig¶s adversary. y New Jersey has added to Rule 1.7 Conflict of Interests with Current Clients an ³Appearance of Impropriety´ standard. o The standard is if a layperson would think there was a conflict of interest. o Purpose of this Rule: Bolster public¶s confidence in the legal system.

y

y

o The Court found that disqualification is compelled in this case because there was an appearance of impropriety. Rule 1.9 (a) ± A lawyer who has formerly represented a client in a matter shall not thereafter represent another person in the same or a substantially related matter in which that person¶s interests are materially adverse to the interests of the former client unless the former client gives informed consent, confirmed in writing. o Once it has been established that an attorney has undertaken a prohibited representation under 1.9, the NJ Courts apply a presumption that confidential information has been revealed. o This presumption may not be rebutted for fear that the former client will be damaged by disclosure of the very information for which the protection is sought. The Court found that these rules, read together, make it clear that the representation of Weiner by Amdur falls within the scope of transactions that were intended to be prohibited by the Rules.

y

RATIONALE: Court found that a conflict of interest has occurred in this situation that compels the Amdur firm be disqualified from representing both parties (Greig and the D). The concerns which require disqualification outweigh Weiner and Macy¶s right to counsel of their choice. IMPUTED DISQUALIFICATION AND MIGRATORY LAWYERS o Rule 1.10 operates from the premise that a firm of lawyers is essentially one lawyer for purposes of the rules governing loyalty to the client, or from the premise that each lawyer is vicariously bound by the obligation of loyalty owed by each lawyer with whom the lawyer is associated. o The Model Rules DO NOT PERMIT SCREENS when private lawyers change firms, although they do permit them when government lawyer enter private practice. j

y

 Session 9.1
 Cromley v. Board of Education  FACTS: Cromley (P) brought and action against the Board of Education (D), claiming she had been denied various administrative positions because she had complained to a state agency about the sexual misconduct of a co-worker. During pre-trial litigation, P¶s attorney, Weiner, accepted a partnership in the law firm representing the D. Weiner then withdrew as P¶s attorney. The District Court granted SJ to the D, denying P¶s motion to disqualify the D¶s attorneys. P appealed. RULE: When a lawyer in a case moves to the other parties firm, the attorneys for the other party must be disqualified where the representations are substantially related, unless the presumption of shared confidences can be rebutted.  RATIONALE: In this case the representations are substantially related but the presumption of shared confidences has been successfully rebutted by the timely establishment of a screening process. After he joined the D¶s law firm, Weiner was denied access to the relevant files. Under threat of discipline, he and all employees of the firm were admonished not to discuss the case. In addition, Weiner was not allowed to share in the fees derived from the case. The partner handling the case for the D affirmed under oath that all of the admonitions had been adhered to. o This case stands for the proposition that the presumption of shared confidences has been found to be irrebuttable only when an entire law firm changes sides. o Other factors help to determine whether adequate protection of the former client¶s confidences has been achieved. Those factors include: 1. The size of the law firm 2. The structural divisions of the law firm 3. The screened attorney¶s position in the firm 4. The likelihood of contact between the screened attorney and one representing another party

o

o o

o o o

o

o

o o o o

o o o o

5. The fact that the law firms and lawyer¶s most valuable asset are their reputations for honesty, integrity and competence. Difference between this case and Analytica: a. This case: Individual Lawyer Switched Sides b. Analytica: Whole Firm Switched Sides The Model Rules disagrees with this ruling that allows the use of screening mechanisms to rebut the presumption of shared confidences. Two different presumptions a. Presumption that the attorney acquired information from the first firm b. Presumption that he transmitted Look at this from the giving firmer/attorney/client, and receiving firm/attorney. Proved that they effectively screened the information in a timely manner. How do the rules handle these presumptions? a. Under the rules, if the attorney possesses the information, then the attorney is conclusively presumed to have spread throughout the new firm. b. What rule # are we concerned with? i. 1.6 ii. 1.10(a) if a lawyer is prevented from representing a client based on 1.9, and 1.9 says they cant iii. 1.9 is the basis for the decisions. If we look at 1.9, then we proceed to 1.10 iv. 1.9c: where does 1.9c say that screening is impermissible but that there is a rebuttable presumption? v. 1.9b2 says the lawyer cannot in a subsequent firm represent a prior client if htat lawyer had acquire confidential information, but if he did not acquire this information, there is no problem. What 1.9b says there is a question did the lawyer at firm 1 acquire pertinent confidential info, and if he did, then in the subsequent firm the lawyer cannot represent a person against a former client. Here there is no question about the confidential information. If he had not then the subsequent firm would be ok. But once it is found that he acquired that pertinent information, that¶s it, the lawyer shall not knowingly represent the same person in a substnatilaly related matter (or same matter)« In 1.9(b)(2)«. Can¶t represent clients who are against the original firms clients, but some jurisdictions allow screening requirements to be met (not all of them), unless he does not have confidential info. Purpose of screening is to ensure that a client of a law firm remains confidential. Screening is permitted under certain circumstances under model rules a. Rule 1.11, when someone has been a public officer and has move to a private firm or has confidential governmental information. b. Rule 1.12c c. Rule 1.1a Screening applies to everyone, so long as screening is permitted by the jurisdiction. Screening is a technique that present lawyer mobility between firms. 1.8b has to do with use of information, rather than the disclosure of information, and 1.6 is concerned with disclosure, as is 1.9c2. HYPO; Plaintiff sues physician claiming medmal. Plaintiff is represented by lawyer 1. Physician/ is represented by lawyer 2. Plaintiff loses. He eventually believes his lawyer was incompetent. Going to sue lawyer for legal malpractice to find lawyer to do that. Plaintiff settles on lawyer 2 to pursue medmal claims. a. Under rule 1.9 can the lawyer do it? b. Legal vs. Medical malpractice. i. Lawyer 1 is the defendant in second suit.

ii. So if lawyer 2 accepts the case, he¶s going to have to damage the reputation of his former physician client. Therefore when the rule says µshall not represent a new client against a former client¶ if the interest of the new client is materially adverse to the former client. iii. Why are the two matters substantially related: 1. Say these are the facts that lawyer 1 committed malpractice, but underneath that you have the facts of the original case. More importantly, comment 3, a substantial risk that confidential factual information as would normally have been obtained in the prior representation would materially advance the clients position in the subsequent matter. OR IF The Model Rules and Screening: o The MR allow the screening of support personnel and former summer associates. o Screening is not allowed to rebut the presumption of shared confidences.

(GOVERNMENT SERVICE)
 Armstrong v. McAlpin  Allows for Screening of Prior Government Attorney to Avoid Disqualification.  FACTS: The SEC commenced an investigation against McAlpin (D) and others, believing them to have looted the company they controlled. A receiver was eventually appointed to attempt to recover the company¶s funds. The firm of Gordon Hurwitz was retained by Armstrong (P), the receiver. A new attorney at Gordon Hurwitz was Altman, who had been an attorney with the SEC involved with the original probe. D moved to disqualify GH, contending that the presence of Altman constituted a conflict. The district court, after receiving testimony that Altman was screened form ongoing litigation, denied the motion. A Second Circuit panel reversed, holding that government service per se disqualifies a fim which included a former gov. attorney from handling an action in which the attorney had previously been involved. The Second Circuit held a rehearing en banc.  RULE: It is not per se grounds for disqualification that an attorney involved in a government investigation joins a private firm involved in litigation concerning the same matter. RATIONALE: Under Model Rule 1.10 the disqualification of one firm member disqualifies the entire firm. However, policy reasons exist for not applying this rule when the cause of the conflict is prior government service. WHY? If it were applied it would be extremely difficult for the government to obtain qualified lawyers to work for it, as they would face the prospect of being unable to obtain private employment ever again. Rather, the better review is to consider disqualification on a case-by-case basis. If a court finds the prior government attorney to be effectively screened from the private litigation, disqualification is not necessary. Here, that was the holding of the district court, and it was proper ruling. j 1.11a says that a lawyer who has formally served as a public officer or employee of the government  1. Is subject to rule 1.9(c)  Shall not otherwise represent a client in connection with a matter in which the lawyer participated personally and substantially as a public  Personal and substantial participation are two separate questions. j 1.11c«a lawyer having information which is confidential gov¶t information acquired while a public servant«shall not j 1.11c does permit screening (last sentence). y

y  Board of Overseers v. Whalley y FACTS: Attorney Whalley (D) represented a mother in a DHS hearing where he learned that she had substance abuse problems. The case culminated with custody of the mother¶s children being given to the father and closure of the D¶s case file in February 2003. In August 2002, D commenced representation of Huck who sought D¶s help in making her husband comply with the terms of the divorce judgment. At this time, D had no knowledge of any conflicts of interest. Huck became concerned that the ex-husband¶s girlfriend was having contact with Huck¶s children during visitation. D retained a temporary protection order for Huck. At this time, D became aware that the ex-husband¶s girlfriend was the mother from the DHS hearing, a former client. D advised Huck that he could only handle the matter if it did not proceed to a hearing. D said he knew the attorney representing the ex-husband and was confident the case could be settled without reaching trial. D neither sought nor obtained he permission of his former client to represent Huck in this matter. Later in 2003, after hearing that the ex-husband planned to marry the girlfriend (his former client) who was now pregnant, the D wrote a letter to the counsel for the ex-husband. In the letter, D disclosed that eh was involved in the former clients DHS matter, and that he expected that DHS would not allow the woman to keep an infant child. D¶s former client filed a complaint with the Board of Overseers of the Bar (P) alleging that the letter violated her expectation of confidentiality and revealed a conflict of interest. RULE:  The primary purpose of attorney discipline is not to impose punishment, but to protect the public. y Punishment here is only a reprimand because Whalley made the wrong decision in a close call.  Model Rule 1.9 Duties to Former Clients y (a) Lawyer should not represent a client whose interests are materially adverse to a former client unless the former client gives informed consent, confirmed in writing. j Court found that D¶s actions were adverse to his former client and that he failed to receive informed consent.  Model Rule 1.6 Confidentiality of Information y Need informed consent to reveal confidential information of client. j Court found that D failed to get informed consent. j The Rule used in Maine covers information obtained in the course of representation that would be ³embarrassing or detrimental to the client.´  The Court found this information to be such.  Rule 1.6 comment 2. Does this include µembarrassing¶ information? Not explicitly. Only legally damaging information is. IN any event, it depends on how the state construes its rules. j   Colorpix Systems v. Broan MFG-- Is Representation of Parent Company and Affiliate Company and then Representing a Client Against another Subsidiary of them a Conflict of Interest?  FACTS: Parent company of Broan (D) was Nortek. Both business¶ manufacture various types of air handling units. They share a legal department, strategy in defending such cases, corporate boards, philosophy, etc. Nortek in 1997 hired R&C as counsel to defend them in a lawsuit stemming from an allegation that one of Nortek¶s air handlers started a fire. In 1998, Colorpix (P) hired R&C in a lawsuit against Broan. RULE: * HOLDING: R&C is disqualified from representing P. The Court held that: (1) sufficient indicia of "vicarious" attorney-client relationship existed to trigger conflict-of-interest inquiry; (2) issue in prior subrogation action, i.e. alleged defect in heating and cooling unit, was substantially related to issue in instant action, i.e. alleged defect in bathroom exhaust fans; and

(3) there was likelihood of access by insurer's law firm to privileged information that would disadvantage manufacturer. RATIONALE: y Attorney may be disqualified from representing client in particular case if: (1) moving party is former client of adverse party's counsel; (2) there is substantial relationship between subject matter of counsel's prior representation of moving party and issues of present action; and (3) attorney whose disqualification is sought had access to, or was likely to have had access to, relevant privileged information in course of prior representation of client. j  Legal Ethics Opinion 1794 of the Virginia State Bar y There is consultation y 1.17 y What about a disclaimer? Can attorney A have sign an effective disclaimer to employ confidential information. y What is informed consent? Rule 1.0E, pg 7, which says that informed consent occurs after an adequate communication and explanation«. y

Session 9.2 NEGOTIATION AND TRANSACTIONAL MATTERS 
 Rubin v. Schottenstein, Zox & Dunn  FACTS: Rubin (P) and Cohen were contemplating investing a substantial sum of money in MDI, a company. Rubin (P) and his attorney both contacted Barnhart, the attorney of Schottenstein, Zox & Dunn (D) who represented MDI. MDI was in default to star Bank, its principal funding source, and an investment by Rubin (P) would constitute another default. Both MDI and Barnhart failed to disclose this information to Rubin (P) and his attorney. In fact, Barnhart assured Rubin (P) that MDI was not having any problems with Star Bank and not to contact the bank directly. Rubin (P) subsequently invested in MDI, Star Bank immediately froze its account, MDI declared bankruptcy, and Rubin (P) lost his entire investment. Rubin (P) filed suit against Schottenstein, Zox & Dunn (D), alleging securities law violations, fraud, and constructive fraud.  RATIONALE: Thos individuals who have direct contacts with the third party have a duty to disclose material facts. Attorneys representing clients who wish to sell securities to those third parties often have such direct contact. The conversations Barnhart had with Rubin (P) were clearly examples of ³direct contact´ and rose to the level that would trigger Barnhart¶s affirmative duty to reveal all the information. The reasonableness of Rubin¶s (P) reliance on Barnhart¶s misrepresentations and omissions may be presumed, particularly because the defendants failed to introduce any evidence to rebut such presumption.  RULE: The status of being an attorney does not confer immunity from liability if the attorney aids a client in actionable or illegal conduct. y  Section 10(b) and Rule 10(b)-5 of the Securities Exchange Act

 Session 10.1
 The Florida Bar v. Belleville  FACTS: Walter Belleville (D), a Florida attorney, was retained as counsel for Bradley Bloch to negotiate an agreement with James Cowan to purchase property owned by the latter. Cowan was eighty-three years of age and had a third-grade education. The various written agreements overwhelmingly favored Bloch. Although Cowan and Bloch had negotiated only for the sale of an

apartment building, the documents stated that Cowan was selling both the apartment house and his residence, which was located across the street from the apartments. The referee found that Cowan had no intention of selling his residence and did not know that it was included in the sale. It was unclear whether Belleville (D) knowingly participated in his client¶s activities or merely followed the client¶s instructions without question. Whatever the case, Belleville (D) drafted the documents to include Cowan¶s house in the sale, and Cowan signed the documents without realizing he was transferring title to his house. A referee recommended no discipline against Belleville (D) on the basis he owed no attorney-client obligation to Cowan. The Florida Bar (P) appealed.  RATIONALE: An attorney has a duty to explain to an unrepresented opposing party the fact that the attorney is representing an adverse interest and must explain the material terms of the documents involved. The record established that Cowan had negotiated to sell the apartment, that he did not intend to sell anything other than the apartment, and that he did not know that the documents of sale would result in the loss of his residence. Furthermore, Belleville (D) should have harbored suspicions about the documents he was preparing because the documents established on their face a transaction so one-sided as to put Belleville (D) on notice of the likelihood of their unconscionability. When the transaction is as one-sided as that in the instant case, counsel preparing the documents is under an ethical duty to make sure that an unrepresented party understands the possible detrimental effect of the transaction and the fact that the attorney¶s loyalty lies with the client alone. Here Belleville (D) breached that duty. The no discipline recommendation of the referee is reversed.  RULE: An attorney has a duty to explain to an unrepresented opposing party the fact the attorney is representing an adverse interest and must explain the material terms of the documents involved. y  Fire Insurance Exchange v. Bell  FACTS: Jason Bell (P), an infant, was severely burned by leaking gasoline at his grandfather¶s apartment. His guardian sued the insurer of the apartment. Robert Collins was retained to represent Bell (P) in his insurance claims. During negotiations, counsel for the insurance company intentionally made material misrepresentations to Collins as to the limits of insurance coverage, stating the limits to be $100,000 when actually the limits were $300,000. Collins subsequently informed Bell (P) he had intentionally been deceived by the opposing insurance counsel, whereupon Bell (P) brought suit against Fire Insurance Exchange (D), the insurer, for its attorney¶s fraudulent misrepresentation of the policy limits. Fire Insurance Exchange (D) moved for summary judgment arguing there was no right to rely on its attorneys¶ representations. Motion denied, Fire Insurance Exchange appealed.  HOLDING: An attorney has the right to rely upon material misrepresentations made by opposing counsel. Courts have a particular constitutional responsibility with respect to the supervision of the practice of law. The reliability and trustworthiness of attorney representations constitute an important component of the efficient administration of justice  RULE: An attorney has the right to rely upon material misrepresentations made by opposing counsel j  Virzi v. Grand trunk Warehouse and Cold Storage Co.  FACTS: Virzi (P) sued Grand Truck Warehouse & Cold Storage Co (D) for personal injuries. A mediation was held. Unbeknownst to all involved, Virzi (P) had died of causes unrelated to the case several days prior to the mediation. The mediator valued the case at $35,000. Grand Truck (D), on counsel¶s advice, accepted the figure. A hearing to confirm the settlement was held. Prior the hearing, Virzi¶s (P) counsel became aware of his death. He did not reveal this fact at the hearing, although he was not so asked. The district court confirmed the settlement. Grand Truck¶s (D) counsel learned of Virzi¶s (P) death and moved to set aside the settlement.  HOLDING: Counsel for a deceased party must inform counsel and the court of his client¶s demise if such death is relevant to the litigation. While an attorney is under a clear duty not to make false

asserts of fact to the court and opposing counsel, an attorney¶s duty of candor is higher than that. This candor involves a requirement of disclosing information essential to the merits of a suit, whether asked about the relevant facts or not. Whether or not Virzi (P) was alive was certainly a matter that went to the very essence of the value of the case, and counsel¶s failure to reveal Virzi¶s (P) demise constituted a sharp practice that cannot be condoned.  RULE: Counsel for a deceased party must inform counsel and the court of his client¶s demise if such death is relevant to the litigation. j  Dietz v. Doe  FACTS: Auto accident caused when car collided with another truck when he lost control trying to avoid another car by making an unsafe, left turn. P died. Driver of the vehicle who made an unsafe, left turn left the scene and contacted attorney Ritchie but the identity of him was kept confidential and attorney Ritchie refused to disclose the identity of the driver claiming asserting that the communication was protected under the attorney-client privilege. No one else knew the identity of the car which made the unsafe left turn. Driver was named ³Doe.´  HOLDING: Attorney-client privilege must be balanced against benefits to administration of justice stemming from a general duty to give what testimony one is capable of giving. Attorney-client privilege exists to allow the client to communicate freely with attorney without fear of compulsory discovery. In this case, there is an inadequacy on the record on whether an attorney-client privilege existed. o Model Rule 4.1 Truthfulness In Statements To Others o In the course of representing a client a lawyer shall not knowing  (a) make a false statement of material fact or law to a third person; or  (b) fail to disclose a material fact when disclosure is necessary to avoid assisting a criminal or fraudulent act by a client, unless disclosure is prohibited by Rule 1.6. o Comment 1 ± omissions can be the equivalent of affirmative false statements o Model Rule 1.2 Scope of Representation and Allocation of Authority Between Client and Lawyer o (d) A lawyer shall not counsel a client to engage, or assist a client, in conduct that the lawyer knows is criminal or fraudulent, but a lawyer may discuss the legal consequences of any proposed course of conduct with a client and may counsel or assist a client to make a good faith effort to determine the validity, scope, meaning or application of the law.  Lawyers must withdraw from representation from representation if he learns that his client is engaged in a crime or fraud in connection with the suit that the lawyer is involved with o Model Rule 8.4 Misconduct o (d) It is professional misconduct for a lawyer to engage in conduct involving dishonesty, fraud, deceit or misrepresentation o Model Rule 4.4 Respect For Rights of Third Persons o (a) In representing a client, a lawyer shall not use means that have no substantial purpose other than to embarrass, delay, or burden a third person, or use methods of obtaining evidence that violate the legal rights of such a person 

 Session 10.2: LAWYERS FOR ENTITIES  CONFLICTS AND CONFIDENTIALITY IN ENTITY REPRESENTATION

 Purpose of rule 1.6 is to tell us what we can tell others, and to protect attorney from disclosure requested by court.  Tekni-Plex, Inc v. Meyner & Landis





 

FACTS: Tom Tang was the president, chief executive officer, and sole director, and sole shareholder of Tekni-Plex, Inc. Meyner and Landis (M&L) (D) represented Tang and Tekni-Plex on various legal matters, including securing an environmental permit for the operation of a laminator machine. Tang agreed to sell Tekni-Plex to the TP Acquisition Company (Acquisition). Acquisition was a shell corporation whose sole purpose was for the purchase of Tekni-Plex. Once the two corporations are merged, Tekni-Plex ceased to exist and Acquisition would change its name to Tekni-Plex, Inc. (new Tekni-Plex) (P). Tang represented that old Tekni-Plex was in full compliance with all applicable environmental laws and possessed all requisite environmental permits. Tang and old Tekni-Plex was responsible for indemnification for any losses incurred by Acquisition as a result of misrepresentation. Following the merger, new Tekni-Plex (P) claimed that a laminator machine emitted volatile organic compounds and was therefore not allowed to operate. New Tekni-Plex (P) brought a claim against tang for indemnification and Tang retained M&L (D) as counsel. New Tekni-Plex (P) sought to enjoin M&L (D) from representing Tang. HOLDING: Counsel cannot represent a present client against a former client involving matters that are substantially related to the prior representation and where the interests of the present client are materially adverse to the interests of the former client. New Tekni-Plex had the burden of satisfying a three pronged test for the disqualification of M&L RULE: Counsel cannot represent a present client against a former client involving matters that are substantially related to the prior representation and where the interests of the present client are materially adverse to the interests of the former client.

j  Murphy & Demory, LTD., Et al. v. Admiral Daniel J. Murphy, U.S.N (Ret.), Et al.  FACTS: Admiral Murphy (D) and Demory (P) co-owned Murphy & Demory, Ltd. (P). The law firm for Murphy & Demonry, Ltd. (P), Pillsbury, Madison & Sutro (D), through its attorney Siemer (D) and Mendelson (D) assisted Murphy (D) in his efforts to take control of Murphy & Demory (P) or to form, before resigning from the company, a new corporation to compete with Murphy & Demory (P). Pillsbury (D) ignored its junior associate¶ warnings that the dual representation was rife with conflicts of interest, with possible breaches of fiduciary duty, and use of corporate opportunities. Murphy & Demory (P) filed suit against Admiral Murphy (D) and against Pillsbury (D).  HOLDING: Where dual representation of both a corporation and its individual owners presents a conflict of interest, the attorneys must obtain the corporation¶s consent for such representation after full disclosure of all material facts. Here, Pillsbury (D) failed to disclose the conflict, to obtain consent for the dual representation of both Murphy (D) and Murphy & Demory, Ltd. (P), or, failing that, to withdraw from the representation. In concluding that there was no conflict, Siemer (D) willfully ignored the Rules of Professional Conduct. As a direct and proximate result of Pillsbury¶s (D) legal malpractice, Murphy & Demory, Ltd. (P) suffered compensatory damages in the amount of $500,000.  RULE: Where dual representation of both a corporation an its individual owners presents a conflict of interest, the attorneys must obtain the corporation¶s consent for such representation after full disclosure of all material facts. y

 Session 11.1
 Fassihi v. Sommers, Schwartz, Silver, Schwartz & Tyler  FACTS: Shareholder and officer in a professional medical corporation brought action against corporation¶s attorney alleging breach of attorney-client relationship, breach of fiduciary, legal, and ethical duties, fraud and legal malpractice stemming from plaintiff¶s ouster from corporation. 2 radiologists (Fassihi and Lopez) joined together to form a partnership and were represented by 1 lawyer. Same lawyer represented Lopez to help Lopez in terminating Fassihi¶s interest in the partnership.

  

How could the confidential relationship arise? If dr. lopez had worked w/ law firm to deprive dr fasihi of this business arrangement

Under rule 1.13(g), notice that 1.7 isincorporated by reference. ³if the organizations consent to the dual representation is required by Rule 1.7, the consent shall be given by an appropriate official ofhte organization other than the individual who is to be represented, or by the shareholders´  Since rule 1.7 is incorporated by reference in 1.13(g), consent is required if 1.7 requires it. Would 1.7 require it--?  Applicable  Rule 7.1: a lawyer shall not make a false or misleading communication abou the lawyer or the lawyer¶s services. A communication is misleading if it contains a material misrepresentation of fact or law or omits a fact necessary to make the statement considered as a whole not materially misleading.  Rule 4.1(b): failure to disclose a material fact when disclosure is necessary to avoid assisting a criminal or fraudulent act by a client, unless disclosure is prohibited by Rule 1.6.  Does rule 1.6 permit disclosure?  Rule 1.13(f): basically that a lawyer has a duty when dealing with shareholders or constitutions j Fasihi falls under at least 3 of those categories.  Rule 4.3: does this apply? Maybe. Notice comment 1: In the fourth line: ³in order to avoid a misunderstanding a lawyer will typically need to identify the lawyer¶s client´ is this dr. lopez? Yup. y  Douglas v. DynMcDermot Petroleum Operations  FACTS: Following her termination as in-house counsel, black female attorney brought suit against her employer alleging retaliation in violation of Title VII. Attorney when bringing Title VII action disclosed informally to third parties information relating to interoffice complaints of discrimination.  RULE: Once an attorney violates the ethical rules, the attorney loses the privilege to sue. Once ethical rules are violated, the employer is insulated from any liability irrespective of whether it took adverse employment action because the conduct constituted a breach or because the conduct was in opposition to discriminatory employment practices.  Applicable Rules  Model Rule 1.13 Organization As Client y (a) A lawyer employed or retained by an organization represented by the organization acting through its duly authorized constituents y (b) If a lawyer for an organization knows that an officer, employee or other person associated with the organization is engaged in action, intends to act or refuses to act in a matter related to the representation that is a violation of a legal obligation to the organization, or a violation of law that reasonably might be imputed to the organization, and that is likely to result in substantial injury to the organization, then the lawyer shall proceed as is reasonably necessary in the best interest of the organization. Unless the lawyer reasonably believes that it is not necessary in the best interest of the organization to do so, the lawyer shall refer the matter to higher authority to the organization, including, if warranted by the circumstances to the highest authority that can act on behalf of the organization as determined by applicable law y (c) Except as provided in paragraph (d), if j (1) despite the lawyer¶s efforts in accordance with paragraph (b) the highest authority that can act on behalf of the organization insists upon or fails to address in a timely and appropriate manner an action or refusal to act, that is clearly a violation of law, and j (2) the lawyer reasonably believes that the violation is reasonably certain to result in substantial injury to the organization

Then the lawyer may reveal information relating the representation whether or not Rule 1.6 permits such disclosure, but only it and to the extent the lawyer reasonably believes necessary to prevent substantial injury to the organization y (g) A lawyer representing an organization may also represent any of its directors, officers, employees, members, shareholder, or other constitutents, subject to the provisions of Rule 1.7. If the organization¶s consent to the dual representation is required by Rule 1.7, the consent shall be given by an appropriate official of the organization other than the individual who is to be represented, or by the shareholders. What did DynMcDermott do? they were operating the strategic oil reserves. Where did the problem arise? She was handing out the letter. This whistleblower was working for DOE, but not DynmcDermott. Duty of loyalty goes hand in hand with duty of confidentiality. We have a lawyer here who believes she¶s a victim of race and sex discrimination in which she makes the race and sex allegations, and adds to that letter a case involving another corporation and a fellow employee, and some of those employees received this property of the company She¶s suing for wrongful termination What duty did she breach? Duty of confidentiality. It did not fall within exception of 1.6 so she shouldn¶t have disclosed this to an outside source. And the confidential information that she disclosed was not information about her, but about two separate complaints altogether. Also breached the duty of loyalty b/c it goes hand in hand w/ confidentiality Why is loyalty pertinent here? what was involved int eh duty of loyalty that doulas violated? y Title VII prohibits retaliatory firing incident. Also prohibits penalizing employee opposing acts protected by title VII. Was there participation here by douglas in the sanction against her? Was there a violation of Title VII? This is leading to the question of what rights do we give up as a lawyer for working for a corporation. They make a balancing test between Rights and conduct. They say Title VII rights between employee and employer interests and the profession in general. Court identifies three factors in balancing interest of employee against the other interests. 1. Entity interest of the employer. what did court say? Employer has to trust its counsel to know that the employee will maximally represent it. The client (employer) must have a reasonable expectation that its confidentiality will not be disclosed. 2. Interest of the legal profession the profession of law has an interest in having its conduct fulfilled and lived up to. Or else, the legal profession suffers and so do those who lawyers represent. 3. Interest of employee: Has the court effectively denied the plaintiff a title vii claim? Perhaps. What does the court rule as to douglas given to what she in fact did: that her actions are not under title vii b/c she breached her duties, so she lost any rights she had for breaching loyalty and other duties. Notice footnote 16:

 Hirobayashi: counsel tried to put in footnote of brief that there was inconsistencies. It didn¶t work. To the memorandum that said the curfew was unnecessary. He went along and signed it. A year later he¶s working on brief for korematsu, remembers the memorandum of the ONI, becomes suspicious of the army¶s assertions for exclusion orders,



Mr. ennis threatened to resign, and gave the solicitor general pause, and people would begin to ask why he wasn¶t on it. Ennis ultimately signed it though. Assuming the 2003 model rules were in effect. What are the obligations of Mr. Ennis? Who is the client of the attorneys who are preparing this brief? If the client is the people, then does the lawyer have an obligation to reveal these two documents? y Rule 1.13 comment 9. Did Mr. Ennis have an obligation to withdraw? y 1.2(d). and comment 10 y 1.16. a lawyer shall withdraw from representing a client if representing the client will violate the rules of professional conduct or other law. y Is there an intent to deceive the court? Wasn¶t the AG trying to keep the SC from learning about these two documents? That¶s why there were three footnotes. Can a lawyer for the government refuse to even disclose a top secret document? What provisions of the model rules apply? Notice comment 9 to rule 1.13: Government agency. ³When the client is a governmental organization, different balance may be appropriate between maintaining confidentiality and assuring that the wrongful act is prevented or rectified, for public business is involved. In addition, duties of lawyers employed by the government or lawyers in military service may be defined by statutes and regulation;´ the comment is saying we need to adjust our decision according to context. Rule 1.6(b) paragraph 1 or 2. Was there interest by the Japanese? Also, Rule 1.6(b)(6) to comply w/ other law. What does other law cover? Does it cover the DPC? Rule 1.13(b) could he take this to the attorney general or the solicitor general. Would this be violation of law that reasonably might be imputed to organization that might cause substantial harm to that organization. What does procedural due process require? That the tribunal has all of the facts. Might there be a violation of law and would it be imputed to the organization, and would it result in substantial injury to the entity? In comment 9 to 1.13: Rule 3.3 a lawyer cannot misstate a material fact or omit a material fact. Does 3.3(a) apply? Notice the comments. y

 

  

   

   

 Session 11.2
 The Saga of Albinex Video  Did lawyer have an obligation to tell doctor who he represented? 1.13(f) He did know that the organizations ARE adverse to those of the constitutions. He reasonably should have known. Rule says µare adverse¶ what he should have known or did know, that the company has an adverse interest y Was lawyer under an obligation under model rules under an obligation to explain his role to the doctors? when he walked into office he said ³I¶ve been asked to do this as the company¶s lawyer¶ so he¶s saying he¶s representing lawyer. Did he have an obligation to warn doctor further? (See rule 1.13 comment 10 and 11) Comment 10 suggests that lawyer should tell the doctor he should get independent legal counsel. Would dr have turned over files? y Look at Rule 4.3 2nd sentence.





 

y 4.1: y Fraud intent to deceive issue? did it have a purpose to deceive? y 1.6(a) y 7.1. the wording is not limited to advertising. If the answer to the first question is µyes¶, what should the lawyer have said to the doctor specifically if the lawyer had an obligation under the rules? If there is an obligation to warn, how broad must that warning be? notice in 1.13(f), the rule says, ³in dealing with an organizations members, a lawyer shall explain the identity of the client when the lawyer knows or reasonably should know that the organization¶ interests are adverse to those of the constituents with whom the lawyer is dealing what does µexplain¶ require. How far? 3rd issue: did partner, the boss of lawyer, did he do anything wrong that would excuse a violation of the rules by the lawyer? 5.1(b) did the supervisor violate this rule? y Did the supervisors statement that the lawyer should get as much information as possible w/ disclosing as little as possible. y 5.2 doesn¶t matter, the lower lawyer still must adhere to rules. y If we¶re having a tough time finding the lawyer as violating rule, maybe it¶s arguable about the supervisor has not as well. y 1.4(a) lawyer shall keep the client reasonably informed of the matter? y 1.4(b) a lawyer shall explain consult w/ client about the means by which the client¶s objectives are to be accomplished. Did lawyer have an obligation to disclose to the company the health problems that doctor had revealed in interview? What happens now? If the lawyer delivered material to supervisor who went to president, did supervisor have an obligation; could he have gone to the chairman of the board? A higher authority? y The CEO¶s background is in science, not in finance or marketing. Is the decision being made by supervisor a business decision or decision w/in scope of representation of the corporation? Notice Rule 2.1: ³in representing a client, a lawyer shall exercise independent professional judgment and render candid advice. In rendering advice, a lawyer may refer not only to law but to other considerations such as moral, economic, social, and political factors, that may be relevant to the client¶s situation.³ y If you juxtapose 1.13(b) and 2.1, where do we come out? Do they constrain or expand the scope of one another? One can argue that 1.13(b) was put into the rules to apply to lawyer representing entity, but not sure that this will be the way its construed; but keep in mind 2.1. 

 Session 12.1
 The Saga of Albinex Video video cont¶d  Did Baskin do anything that would have violated the rules?  5.1: cmt 3.Last sentence in comment 3: the ethical atmosphere can influence conduct of all employees. Cannot assume that all lawyers will adhere to all the rules of professional conduct. But see 5.2(a) and 5.2(b) we have an ambiguity with the FDA regulations. Delay of report or is it necessary to be quickly filed?  Client wants an opinion letter«.does not have to be transmitted immediately to the FDA. That can be put into a report later on. In that opinion letter, what should the law firm say?  Should it disclose the differences of opinion? Should it be delayed? What should be in this opinion?  1.4(b) + comment 5.  What is the responsibilities of the gov¶t lawyer who receives the letter? What do the rules require him to do?  If the FDA attorney looks at it and realizes there is something wrong, should she find out what it is, or are the FDA regulations not clear enough argue for concluding the gov¶t attorney can wait for the periodic report. Rule 1.3cmt1.



 Session 12.2: DISCIPLINE
 In choosing a sanction, the ABA model standards identify four considerations  The duty violated  Lawyer¶s mental state  Potential or actual injury caused by lawyers misconduct  Existence of aggravating or mitigating factors.  In re. Warhaftig  FACTS: Attorney Warhaftig (D), faced with serious financial pressures, began appropriating client funds. For the most part, the funds were returned. When the State Bar (P) discovered Warhaftig¶s (D) activities, it instituted disciplinary proceedings. Warhaftig (D) argued in his defense that he had only meant to ³borrow´ the monies taken, not permanently misappropriate them. Based on this distinction, the State Bar (P) recommended a reprimand but not disbarment.  HOLDING: An attorney may be disbarred for taking fee advances out of client funds, even though he did so with the intention of returning the funds. An attorney who knowingly misappropriates client funds must be disbarred. Misappropriation is prohibited whether or not the funds were ³stolen´ or ³borrowed.´  RULE: An attorney may be disbarred for taking fee advances out of client funds, even if he did so with the intention of returning the funds.  The attorney, Warhaftig, took funds from the trust account he had set up for clients at a point in time before he had any legal right to such money;  The court said he was effectively borrowing from one group of clients to pay himself in advance for matters being handled for other clients.  Warhaftig argues that he never failed to make a proper payment and nobody ever lost money. y Said he knew what he was doing was wrong, but made sure nobody got hurt  Lower Disciplinary Board emphasized that there was a distinction between knowing misappropriation of clients funds (grounds for disbarment) and what respondent did, which was pre-mature withdrawal of money he had earned.  Rule  A lawyer¶s subjective intent, whether it to µborrow¶ or steal, is irrelevant to the determination of the appropriate discipline in a misappropriation case.  Any attorney who knowingly misappropriates funds will be disbarred.  Analysis  The policy is that we hold lawyers to a higher standard of professional responsibility  Note about what would happen to bank tellers not being rehired  The difference between stealing and borrowing is negligible.  Respondent knew with full recognition that he was making these withdrawals in violation of rules governing attorney conduct.   In re. Austern  FACTS: Austern represented one Viorst, who was attempting to arrange a condominium conversion. At one point he attended a meeting between Viorst and prospective purchasers. The purchasers, suspicious of Viorst¶s motives, demanded that he make a good faith money deposit into an escrow account. Viorst wrote a check for $10,000. Viorst later admitted to Austern that the check was drawn on an account which contained no funds. Despite this knowledge, Austern helped effect the transaction. He was later investigated by the Board on Professional Responsibility, which recommended censure for his part in the deal.  HOLDING: An attorney may be censured for facilitating a real estate transaction despite knowledge that a client¶s deposit to third parties consisted of a check drawn on nonexistent funds. Viorst¶s use

 

of a worthless check as earnest money constituted fraud on the prospective purchasers. Auster, with knowledge of this fraud, helped facilitate the transaction. The only appropriate course of action for Austern would have been to advise Viorst to cease, and upon Viorst¶s failure to do so, withdraw. RULE: An attorney may be censured for facilitating a real estate transaction despite knowledge that a client¶s deposit to third parties consisted of a check drawn on nonexistent funds. 8.4 B,C,D Austern: 4.1 comment 1: partially true but misleading statements 1.16a i. A lawyer shall not assist the client with conduct that is fraudulent (8.4c) ii. Lawyer should abide by clients wishes (1.2a) but should not assist client in engaging in illegal conduct (1.2c); if clients wishes are to engage in fraudulent conduct, lawyer should withdraw from representation (1.16a1) 1. Court: respondent was under an affirmative duty to withdraw from representation of Viorst once he knew that the escrow account was funded with a worthless cheque.

 In re. Martha Jane Shay y FACTS: Attorney represented J.C. J.C. married L.F.C. who later became legally separated. J.C. and E.Y. began to date and told E.Y. falsely that the divorce was final with respect to him and L.F.C. but J.C. legally remained married to L.F.C. Attorney knew that J.C. was still legally married to L.F.C. Attorney drafted wills for E.Y. and J.C. which E.Y. was unaware of the fact that J.C. was still legally married. J.C. incurred debt and E.Y., upon obtaining a credit report which listed L.F.C. as J.C.¶s spouse found out that J.C. was lying. J.C. asking attorney not to tell E.Y. that the divorce was not final told E.Y. to leave that alone and not to worry about the fact in the credit report.  Whether respondent had a conflict of interest and whether she failed to withdraw after she knew or should have known that her continued employement would result in a violation of the disciplinary rules it is first necessary to determine if she had an attorney-client relationship between both J.C. and E.Y. y y y Applicable Rules Rule 8.4 Misconduct j It is professional misconduct for a lawyer to:  (a) violate or attempt to violate the Rules of Professional Conduct, knowingly assist or induce another to do so, or do so through the acts of another;  (b) commit a criminal act that reflects adversely on the lawyer¶s honesty, trustworthiness or fitness as a lawyer in other respects  (c) engage in conduct that involving dishonesty, fraud, deceit or misrepresentation  (d) engage in conduct that is prejudicial to the administration of justice  (e) state or imply an ability to influence improperly a government agency or official or to achieve results by means that violate the Rules of Professional Conduct or other law; or  (f) knowingly assist a judge or judicial officer in conduct that is a violation of applicable rules of judicial conduct or other law. Rule 1.2 Scope of Representation and Allocation of Authority Between Client and Lawyer j (d) A lawyer shall not counsel a client to engage in, or assist a client, in conduct that the lawyer knows is criminal or fraudulent «

y

Rule 4.1 Truthfulness in Statements to Others j (a) In the course of representing a client a lawyer shall not knowingly make a false statement of material fact or law to a third person y Court: Shay cannot place her duty on J.C. to cure her conflicts of interest y Conclusion: violation of 1.7/1.16\ Shay can continue representing E.Y. so long as it is obvious that she can adequately represent her after full disclosure of the possible effects of representation. (But she never made the disclosure, so this isn¶t up for discussion: y Violation of Rule 1.7(b)(2)  Notwithstanding the concurrent conflict of interest between E.Y. and J.C., Shay may still represent client if the lawyer reasonably believes that she will be able to give competent representation to each client, it isn¶t prohibited by law, does not involve the assertion of a claim by one client against another, and each affected client gives informed consent, (in writing)  Court  There was a conflict of interest with regards to the Bank Loan  Shay never properly advised E.Y. with regard to the loan and never corrected the belief that she was married to E.Y.  Critical fact: EY and JC were still not married!!  Shay told EY it would be dangerous for reporting the forgery  Once JC knew the signature was forged, the interests of EY and JC were in conflict  Shay left EY open to future litigation  If Shay was not representing JC, she could have reported the forgery, but because she was representing JC, she had to make sure it was not. (particularly b/c there was no legal relationship)  Shay never disclosed this conflict of interest or the consequences of such representation. Respondent engaged in Acts of Dishonesty 1. µfraudulent¶ µdeceitful¶ or µmisrepresentative behavior¶, and encompasses the broader conduct of lacking honesty, integrity, and fairness. a. Drafting EY¶s will satisfies this: i. Perpetuates JC¶s dishonesty (said he was her husband) (also 4.1)«can it be used to interpret 8.4(c) 1. She knew µhusband¶ was a false and misleading term. 2. argues that she did not make a conscious decision to deceive She cannot decide on her own that, with only one client knowing the facts, that representation serves the good of both clients. j

 Session 13.1
 Lawyer Trust Accounts Video Iolta: interest on lawyer trust accounts Rule 1.15 - The ethics rules on trust accounts come from principle and agent, fiduciary and beneficiary - The relationship is intended for contract purposes; apply it to disciplinary rules, and these are your ethics rules. - Lawyer is agent; client is principal. Lawyer becomes fiduciary o Duty to account to the client o Duty to give information o Duty to safeguard property o Duty for confidentiality o Cannot lie to principal

 Anytime you research a trust account question/ethics question, be careful to understand what year/event occurred, the case opinion was decided«all of these change over time. History is important.  Make a list of a  Trust account responsibility is personal, it is not delegable (to managing partner, spouse, secretary«etc..), and includes supervision of others, and includes vicarious liability for others in your office.  Can¶t use ³I was just following orders´  Don¶t ever use a rubber stamp, or mechanically sign. You¶re stuck if you do.  Do not protect you from regular endorsements  In a single client client-trust-account; want to put the money in those accounts, and the client gets the interest and the 10-99 by the gov¶t. Now we take a multiple-client account  Over-draft protection. No cheques ever bounce. Even overdraft protection might not be good enough.

 DO NOT HAVE YOUR MONEY IN CLIENT-TRUST Account  If the money is yours, it must first be taken from this account and put into an office account, and then from there, to a personal account.  This is co-mingling; so don¶t put your money in the trust account.  One way it may happen unintentionally: get a settlement check; deposited; cheque deposited; what have you done? Those cheques that you wrote, bounce, and the money is covered from other accounts; this compounds the problems. This is BAD  You have to know what you¶re putting into the trust account.  Wire transfers are a good idea  Fees and trust accounts  Minimum non-refundable fee is not a minimum and it¶s not non-refundable.  A true µretainer fee¶ is that the instant you¶ve received it you¶ve earned it b/c you¶re available.  Otherwise, it¶s an advanced payment, and it¶s put into a trust account.  More than half the clients want to change lawyers before the case is over.  Client has a right to fire you; and if the money they gave you prevents them from getting  If you¶ve µearned¶ the money, you must take the money out. If the client charges against this, you must put the money back in until the matter is resolved.  Fee Disputes at 10 days (or reasonable amount of time) can be taken out. Otherwise you gotta put the money back in.  Record-Keeping  Must keep trust account records separate. Keep them forever in a fireproof safe.  Do not rely on a computer to keep or maintain records. Get hardcopy:  Unearned fees must stay in the trust account  Recommendation: leaving about 50 bucks of your own money in the account; may get unknown charges for wiring or cc. you have to account for this. Keep some money in there. Be sure to under the FDIC rules  1. Simple bank reconciliation  2. Total money in trust accounts belong to ABCDEF, here¶s how much came in«  3. Reconcile cash of the accounts w/ client list  Put the minimum fee retainer and only take it out if you think you¶ve earned it. j  Rule 1.15 and Comments

 Session 13.2: JUDGES ± CONFLICTS and DISQUALIFICATION
 Liljberg v. Health Services Acquisition Corp.







FACTS: A contractual dispute arose between Liljeberg (P) and Health Services Acquisition Corp. (D) over the right to construct a hospital. Loyola University stood to benefit if Liljeberg (P) won. The judge to whom the case was assigned was a trustee of Loyola, although he apparently was unaware of Loyola¶s interest in Liljeberg¶s (P) success. After a bench trial, the judge announced a verdict in favor of Liljeberg (P). Several days later, before final entry of judgment, Loyola¶s interest was brought to the judge¶s attention. He denied a motion to vacate, and he entered judgment. HOLDING: A judge, upon discovering a personal interest in a litigation, must recuse himself any time before final entry of judgment. A judge shall disqualify himself in any proceeding in which his impartiality might be questioned. While a judge obviously cannot recuse himself for something he does not know, if such knowledge is imparted to him any time during the course of the action, a pall is cast over the whole proceeding, as such important motions as requests for new trials might be made right up to the time of final entry. For a judge to rule on such items with knowledge of a personal interest would undermine public confidence in the judiciary. RULE: A judge, upon discovering a personal interest in a litigation, must recuse himself any time before final entry of judgment. y Applicable Rules j Rule 8.4 Misconduct  It is professional misconduct for a lawyer to:  (c) engage in conduct that involving dishonesty, fraud, deceit or misrepresentation  (d) engage in conduct that is prejudicial to the administration of justice  (f) knowingly assist a judge or judicial officer in conduct that is a violation of applicable rules of judicial conduct or other law. j Rule 3.5 Impartiality and Decorum of the Tribunal  A lawyer shall not:  (a) seek to influence a judge, juror, prospective juror or other official by means prohibited by law  (b) communicate ex parte with such a person during the proceeding unless authorized to do so by law or court order

y  Comm. On Legal Ethics v. Hobbs y FACTS: Lawyer suspended from practicing law for 2 years for extortion. Judge¶s wife was present at hospital where malpractice occurred and judge told lawyer to see wife. Lawyer paid judge¶s wife in exchange for help  Model Judicial Code

 Session 14. 1
 Speaker from Office of Disciplinary Counsel for Supreme Court of Delaware  Model Rules 8.1, 8.3, and 8.5

 Session 14.2: Expressions of Gender, Racial, And Other Bias
 In re Marriage of Iverson y FACTS: The Iversons divorced. George Iverson (D) was an influential individual in the entertainment industry. Cheryl Iverson (P) challenged the validity of an antenuptial agreement she had signed prior to the marriage. Cheryl (P) testified that she had not known what she was signing; George (D) testified that she had been fully informed about the nature of the document. The trial judge held the agreement valid, but did order George (D) to maintain a life insurance policy favoring Cheryl (P). The order was accompanied by an opinion that used some thinly disguised language indicating that Cheryl (P) had been gold-digging in her relationship with George (D), and that she had been little more than a trophy wife to him. Constant references to

y

y y  Matter of Bourisseau  FACTS: During a newspaper interview, Judge Bourisseau (D) was asked for his views on the Parental Rights Restoration Act. Judge Bourisseau (D) expressed his displeasure with the enactment of the Act and stated that he might permit a white minor to have an abortion if she were raped by a black man. Those remarks were widely disseminated in the news media. Several grievances were filed with the Judicial Tenure Commission. Judge Bourisseau (D) indicated it had not been his intention to speak in a racially insensitive manner and expressed regret for his remarks, The Commission recommended an order of discipline. In issuing the order, the court followed the findings of the Commission.  HOLDING: Public remarks made by a judge which are racially offensive and improper constitute misconduct in office. Judge Bourisseau¶s (D) remarks called into question the impartiality of the judiciary and exposed the judicial system to contempt and ridicule. Such erosion of public confidence in the judiciary is clearly prejudicial to the administration of justice. Thus, a public censure is an appropriate response to Judge Bourisseau¶s (D) remarks.  RULE: Public remarks made by a judge which are racially offensive and improper constitute misconduct in office.  MCJC: 2.3 is limited to proceedings.   1.2: applicable to times outside of trial  2.3: within trial   Application: 1.a.b.--> includes all judges and those acting in the position of judges. y  Commonwealth v. Brandenburg

Cheryl¶s (P) physical qualities were made in the opinion, and she was referred to as a ³girl,´ although she was in her forties. HOLDING: A judicial decision cannot be based on stereotypical notions of gender. A trial must be fair and appear to be fair. Acts or words by a judge that show that his ruling is based not on individualized facts but on group notions does not meet this standard. Here, the trial court seems to have cited stereotypical notions of wealthy older men and attractive younger women as the basis for his decision, and this type of stereotyping cannot be condoned. RULE: A judicial ruling cannot be based on stereotypical notions of gender

Sponsor Documents

Or use your account on DocShare.tips

Hide

Forgot your password?

Or register your new account on DocShare.tips

Hide

Lost your password? Please enter your email address. You will receive a link to create a new password.

Back to log-in

Close